*NURSING > TEST BANK > Women’s Health A Primary Care Clinical Guide 5th Edition Youngkin Schadewald Pritham Test Bank,10 (All)

Women’s Health A Primary Care Clinical Guide 5th Edition Youngkin Schadewald Pritham Test Bank,100% CORRECT

Document Content and Description Below

Women’s Health A Primary Care Clinical Guide 5th Edition Youngkin Schadewald Pritham Test Bank Chapter 1 Access to Women’s Health Care in the United States: Affordability, Equity, Rights ... 1. Which health occupation has the highest percentage of women? A. Pharmacists B. Physical therapists C. Registered nurses D. Dental hygienists Answer: D 2. Which health occupation has the lowest percentage of women? A. Physicians B. Dentists C. Pharmacists D. Physical therapists Answer: B 3. Which health profession has the largest number of workers? A. Health aides B. Physicians C. Licensed practical nurses D. Registered nurses Answer: D 4. Which of the following are certifications available in advanced practice registered nursing? (Select all that apply.) A. Certified nurse midwife B. Certified registered nurse anesthetist C. Certified nurse pharmacologist D. Clinical nurse specialist E. Nurse practitioner Answer: A, B, D, E 5. What level of education is required to become a licensed practical nurse? A. 2-year master’s degree (in addition to a 4-year bachelor’s degree) B. 4-year bachelor’s degree C. 2-year associate’s degree D. 1-year certificate or diploma Answer: D 6. What percentage of physicians and surgeons in the United States in 2014 were women? A. 27% B. 37% C. 47% D. 57% Answer: B 7. What medical specialty has the highest percentage of women? A. General pediatrics B. Obstetrics and gynecology C. Orthopedic surgery D. Urology Answer: A 8. On an average, the income of female physicians is what percentage of that of male physicians? A. 59% B. 79% C. 99% D. 109% Answer: A 9. What level of education is required to become a pharmacist? A. 6-year doctorate degree B. 2-year master’s degree (in addition to a 4-year bachelor’s degree) C. 4-year bachelor’s degree D. 2-year associate’s degree Answer: A 10. Which of the following is the median income of dentists in the United States (2012)? A. $89,310 B. $109,310 C. $129,310 D. $149,310 Answer: D 11. Which of the following are the current trends in dentistry? (Select all that apply.) A. More specialists than generalists B. Research linking oral health to overall health C. Focus on treatment of disease rather than prevention D. Expected growth of 18% from 2014 to 2024 E. Increasing demand for dental implants, bridges, and cosmetic services Answer: B, D, E 12. Which allied health occupation is projected to be the fastest growing? A. Health information technicians B. Nursing aides C. Occupational therapists D. Paramedics Answer: A 13. One in five workers in which of the following allied health occupation is at or below the federal poverty level? A. Medical assistant B. Home health aide C. Radiology technician D. Speech-language pathologist Answer: B 14. Veronica is a registered nurse who pours herself into her job. She works long hours without complaint andstrives to do her best. Often, she maintains a cheerful, caring, and kind demeanor on the outside whilefeeling exhausted and frustrated on the inside. The effort it takes to maintain this front before her patientsresults in a lot of stress. This phenomenon can best be described as which of the following? A. Identity crisis B. Psychological disparity C. Emotional dissonance D. Gender discrimination Answer: C 15. Tests of implicit racial bias among health professionals have revealed which of the following? A. An unconscious preference for Whites over Blacks B. Use of racially charged, derogatory language C. Sharing of racist jokes D. Lower pay for racial minorities Answer: A Chapter 2 Women’s Development into the 21st Century 1. To enhance women’s health care in the 21st century, researchers should do which of the following? (Select all that apply.) A. Design studies in collaboration with women B. Analyze changes in women’s health data relative to men’s C. Include homogeneous populations of women in studies D. Translate research findings into clinical and public health practice E. Focus on treatment approaches equally applicable to men and women Answer: A, B, D 2. Historically, gender has been defined by which of the following? (Select all that apply.) A. Self-identification B. Appropriate roles C. Division of labor D. Economic power E. Political influence Answer: B, C, D, E 3. Hammarstrӧm et al. propose a model of sex and gender that includes which of the following concepts? (Select all that apply.) A. Binary sexuality based on one’s chromosomes B. Sex, interacting with gender, as a continuum C. Biologically determined sex based on the effects of sex hormones on reproductive organ development D. Sex and gender as an integration of body, mind, and context E. Intersectionality and embodiment as factors significantly affecting sex and gender Answer: B, D, E 4. Which of the following is the best example of how gender bias has affected the health and health care of women? A. Similar rates of mental illness being found in men and women B. Treatment outcomes among women varying based on patient compliance C. Association of patient income level with type of diagnosis D. Disproportionately more psychotropic medications being prescribed to women than men Answer: D 5. Krieger has proposed which of the following regarding sex, gender, and health? (Select all that apply.) A. Gender and sex played no significant role in determining health outcomes for women B. Gender relations influence the expression and the interpretation of biological traits C. Sex-linked biological characteristics contribute to gender differentials in health D. Traditional perspectives on gender and sex have resulted in better health outcomes for women than for men E. Equitable gender relationships have resulted in similar health outcomes in men and women Answer: B, C 6. When did women’s health scholarship begin to flourish? A. 1960s B. 1970s C. 1980s D. 1990s Answer: B 7. Which of the following were the goals included in the report “Women’s Health Research: Progress, Pitfalls, and Promise,” which was published by the Office of Research on Women’s Health in 2010? (Select all that apply.) A. Increase sex similarities research in basic sciences studies B. Incorporate findings of sex and gender differences in the design of new technologies C. Create strategic alliances to maximize domestic and global impact of women’s health research D. Develop and implement new social networking technologies to promote men’s health and wellness research E. Employ innovative strategies to build a well-trained, diverse, and vigorous women’s health research workforce Answer: B, C, E 8. The National Institutes of Health (NIH) Revitalization Act of 1993 mandated that the NIH do which of the following? A. Expand health insurance coverage for women B. Include women and minorities in clinical research C. Research and develop new women-specific medications D. Decrease infection rates during obstetric surgeries Answer: B 9. Historical examples of gender bias in medical textbooks include which of the following? (Select all that apply.) A. Portrayals of women as inherently sick B. A recommendation that women simulate orgasms if not orgasmic with their husbands C. Portrayals of women patients as being intellectually superior to their male physicians D. Omission of the clitoris from anatomical illustrations of women’s genitals E. Portrayal of women as invincible to illness and age, as long as they make the rightchoices Answer: A, B, D, E 10. Which of the following best describes the new model for health care for women? A. Physician-centered B. Authoritarian C. Pluralistic D. Disease-oriented Answer: C Chapter 3 Epidemiology, Diagnostic Methods, and Procedures for Women’s Health 1:A 46-year-old patient is referred for the biopsy of a vaginal lesion. Vaginal biopsy typically requires what type of anesthesia? a:General b:Caudal c:None d:Conscious sedation e:Pudendal 2:A 36-year-old patient is to undergo removal of her uterus for benign disease. Which of the following is an advantage of abdominal hysterectomy over vaginal hysterectomy? a: Repair of rectocele more readily accessible b: Fewer incisional complications c: Ability to deal with smaller uterine sizes d: Ability to visualize associated pelvic pathology e: Shorter recovery period 3:She undergoes removal of only her uterus. Which term refers to the surgical removal of only the uterine corpus? a:Complete hysterectomy b:Vaginal hysterectomy cTotal hysterectomy d:Subtotal hysterectomy e:Radical hysterectomy 4:A 53-year-old is referred to you for the evaluation of an adnexal mass found at the time of annual physical examination. In the initial evaluation of a possible adnexal mass, the most appropriate imaging technique is a:computed axial tomography scanning b:ultrasonography c:flat plate of the abdomen d:magnetic resonance imaging e:positron emission tomography (PET) imaging 5:What is the most appropriate biopsy location for a 45-year-old woman with abnormal vaginal bleedingfor 5 days and a normal physical examination? a:Vulva b:No biopsy indicated if less than 7 days c:Endometrium d:Vagina e:Cervix 6: A 25-year-old patient had an IUD inserted last year, but no longer feels the string. Which of the following is the most appropriate next step in management? a: Dilation and curettage b: MRI c: Hysteroscopy d: "Flat-plate" X-ray of the lower abdomen e: Ultrasonography 7: A 35-year-old patient does not recall why she had a laparoscopy 3 years ago. Which of the following conditions is most likely to be associated with a laparoscopy for evaluation? a: Pelvic pain b: Abnormal Pap smear c: Abnormal bleeding d: Recurrent cystitis 8: A 32-year-old patient recovering from a gynecologic procedure complains of shoulder pain. The procedure she had was most likely a(n) a: LEEP b: cervical biopsy c: endometrial ablation d: dilation and curettage e: laparoscopy 9: What is the most appropriate initial imaging study in an asymptomatic patient with 18-week-size fibroids? a: Positron emission tomography (PET) b: computed tomography (CT) c: Transabdominal ultrasonography d: Sonohysterography e: Magnetic resonance imaging (MRI) 10: A 20-year-old patient at 6 weeks of gestation is suspected of having an ovarian neoplasm. The most appropriate initial imaging technique is a: Computed tomography (CT) b: Positron emission tomography (PET) c: transvaginal ultrasonography d: transabdominal ultrasonography e: Magnetic resonance imaging (MRI) 11: A healthy, asymptomatic, nulliparous 36-year-old woman requests your advice because she has been unsuccessful at achieving pregnancy over the last 13 months, despite regular menses. Performing an initial physical examination, you detect a firm, non-tender, multinodular uterus whose size corresponds to that at approximately 8 to 10 weeks of pregnancy, which is consistent with leiomyomata uteri. The remainder of the medical history and physical examination is within normal limits, as is her husband's semen analysis. Normal ovulatory status has been documented by basal body temperature monitoring and luteal-phase progesterone measurement. Your next best step is to recommend a: normal pelvic ultrasonography b: hysterosalpingogram (HSG) c: in vitro fertilization - embryo transfer (IVF-ET) d: myomectomy e: arterial embolization of the fibroids 12: A 36-year-old woman with regular, monthly, 5-day menstrual cycles presents with a 2-year history of severe right-sided pelvic pain. Pelvic examination reveals no abnormalities. She reports insomnia, loss of appetite, and decreased libido. The patient once considered evaluation for infertility but now does not desire childbearing. She has had three laparoscopic procedures in the last 18 months that showed minmal peritoneal adhesions, with no change in the pain. Appropriate management of this includes a: repeat diagnostic laparoscopy b: barium enema, intravenous pyelography, and plevic ultrasonography c: presacral neurectomy d: transabdominal hysterectomy and bilateral salpingo-oophorectomy e: psychological counseling with antidepressant therapy 13: A 63-year-old woman with 6 months of early satiety and increased abdominal girth has an abdominal fluid wave and a palpable 12 cm nodular right adnexal mass. The next BEST step would be for her to obtain a: a diagnostic laparoscopy b: a staging laparotomy and debulking for her presumed ovarian cancer c: a CT scan to get the exact dimensions of the mass d: chemotherapy e: an ultrasonography to determine if the mass is cystic or solid 14: A 42-year-old woman, G2P2, who had a laparoscopic tubal fulguration 7 years ago, has abnormal uterine bleeding. Her menstrual cycle has been 28-30 days with a menstrual flow of 5-6 days. During the last 6 months, the interval has varied from 14 to 35 days and the menstrual flow has lasted from 1 to 14 days. The uterus is irregular and firm and consistent in size with an 8-week gestation. Both ovaries are palpably normal in size. Urine pregnancy test is negative. The most appropriate next step in management is a: endometrial sampling b: laparoscopy c: transabdominal ultrasonography d: pelvic CT e: hysteroscopy 15: A 19-year-old primigravid woman is seen because of vaginal bleeding. Her LMP was about 11 weeks ago; the vagina contains a small amount of dark blood and the cervix is long and closed. The uterus is enlarged and not tender. A pelvic ultrasonography demonstrates an intrauterine gestational sac but no fetal pole is identified. The patient is distraught but declines surgical management. Of the following, the most appropriate management of this patient's condition is a: combination oral contraceptive pills b: methotrexate intramuscular injection c: depot medroxyprogesterone acetate d: oral broad-spectrum antibiotic thearpy e: mifepristone pills and vaginal misoprostol ANS: [cddbc eaecc beaae] Chapter 4 Assessing Adolescent Women’s Health Renee Sieving, Sarah Stoddard, Deborah A. Raines 1. The teen birthrate is highest among which major racial or ethnic group? A. Latinos B. African Americans C. Whites D. Asians Answer: A 2. What percentage of the U.S. population was 10 to 19 years old in 2014? A. 14% B. 19% C. 24% D. 29% Answer: A 3. Which of the following are examples of protective factors for adolescents? (Select all that apply.) A. Bonds with friends and family B. Physical inactivity C. A school’s high expectations of students D. Risky sexual behaviors E. Opportunities for youth participation in the community Answer: A, C, E 4. Rachel is a talkative, open 13-year-old who is in for a physical exam today by herself. In talking with Rachel, the nurse should recognize which of the following as characteristics typical of this stage of development? (Select all that apply.) A. Criticism of her parents B. Acceptance of the physical changes of puberty C. Mood swings D. Sexual feelings emerging E. Transition to adulthood Answer: A, C, D 5. Lydia is a teenager who is in the process of researching college programs. She thinks she would like to be a lawyer. She is also searching for a summer job to save up to buy a car. Which stage of development is Lydia most likely in? A. Preadolescence B. Early adolescence C. Middle adolescence D. Late adolescence Answer: D 6. Yancy is a 14-year-old with Down syndrome. She, like many teens with disabilities, is at an increased risk of having which chronic condition? A. Epilepsy B. Obesity C. Heart disease D. Arthritis Answer: B 7. Brooke is a 17-year-old who is questioning both her sexual orientation and her gender. She says she has begun exploring these but that her parents do not know. Brooke is at an increased risk of experiencing which of the following? (Select all that apply.) A. Obesity B. Family violence C. Homelessness D. Substance abuse E. Suicidality Answer: B, C, D, E 8. Rose is a 14-year-old refugee from Nigeria who recently came to the United States. Being a foreign born, which of the following is Rose more likely to do than her native-born classmates? A. Exercise B. Succeed academically C. Eat meals with her family D. Develop positive peer relationships Answer: C 9. Destiny is an outgoing, fun-loving 13-year-old who has developed a strong network of friends at school. These positive relationships make it more likely that Destiny will experience which of the following? (Select all that apply.) A. Academic achievement B. A healthy diet C. Reduced substance abuse D. Improved mental health E. Lack of peer conflict Answer: A, C, D 10. During an office visit, Jordan, a 16-year-old patient, asks her nurse if she can share something in confidence. The nurse affirms, but adds that there were a few disclosures that would legally require breaking confidentiality. Which of the following are issues the nurse would be obligated to report? (Select all that apply.) A. Sexual activity B. Intent to self-harm C. Evidence of abuse D. Gender questioning E. Religious doubts Answer: B, C 11. What percentage of adolescents were covered by health insurance in 2013? A. 48% B. 63% C. 78% D. 93% Answer: D 12. A nurse is having trouble communicating with her teenaged client, Enid, who is disengaged and sullen. What approach would most likely be effective in helping the nurse engage with Enid? A. Giving some concrete goals to Enid for her to pursue B. Asking Enid’s mother to join them C. Using motivational interviewing with Enid D. Reminding Enid of the consequences of noncompliance Answer: C 13. Nadia is a 13-year-old who is in for her first reproductive health visit. She reports having no special concerns and appears healthy. Which of the following is most important for the nurse to do in this visit? A. A Pap exam B. An internal pelvic exam C. An assessment of menstrual issues D. A mammogram Answer: C 14. In a 2013 survey of the Youth Risk Behavior Surveillance System, what percentage of high school females reported drinking more than five drinks in a row on at least 1 day in the 30 days before the survey? A. 16.1% B. 21.1% C. 26.1% D. 31.1% Answer: B 15. A nurse needs to assess an adolescent client’s level of sexual development. Which diagnostic tool should the nurse use? A. Tanner staging B. CRAFFT screening C. Motivational interviewing D. Mini-Cog assessment Answer: A Chapter 5 Assessing Adult Women’s Health Diane Marie Schadewald, Catherine Juve, Ellis Quinn Youngkin, Marcia Szmania Davis 1. Midlife is commonly defined by which of the following ways? (Select all that apply.) A. Age B. Proximity to menopause C. Degree of children’s independence D. Level of cognitive function E. Women’s self-perception of age Answer: A, B, C, E 2. Vivian is an 80-year-old patient who is reminiscing about her own midlife with her nurse, who has just entered midlife. Given her age, Vivian is most likely to identify which of the following as having been a primary source of meaning during her midlife? A. Her career B. Her family C. Her friends D. Her artistic accomplishments Answer: B 3. Which of the following is the definition of menopause currently used in the health sciences? A. The date of onset of menopause-related symptoms B. The cessation of menses, defined as the end of the final menstrual period C. The cessation of menses, defined as 1 year after the final menstrual period D. The date of the last menopause-related symptom Answer: C 4. Women of which U.S. ethnic group are most likely to describe menopause as “something you have to go through” and to view it as a time to reorient and restructure their lives? A. African Americans B. Japanese Americans C. European Americans D. Urban Latinas Answer: D 5. Which of the following endocrine changes are typical during the last 2 years before the final menstrual period? (Select all that apply.) A. Increased testosterone levels B. Decreased antral follicle count C. Increased follicle-stimulating hormone levels D. Decreased progesterone levels E. Increased estradiol levels Answer: B, C, D 6. Sleep disruption, a common symptom during the menopausal transition, is most often associated with which other perimenopausal symptom? A. Hot flashes B. Decline in memory C. Back pain D. Depressed mood Answer: A 7. Which of the following are common symptoms during the menopausal transition? (Select all that apply.) A. Difficulty concentrating B. Urinary incontinence C. Vaginal dryness D. Increased sexual desire E. Nausea and vomiting Answer: A, B, C 8. Which of the following statements is most accurate concerning symptoms of the menopausal transition? A. Almost all women experience the same set of symptoms, known as menopausal syndrome. B. Almost all women experience hot flashes, but other symptoms are too variable to predict. C. Symptoms experienced appear to vary from culture to culture, and thus may be culture bound. D. Symptoms are completely random, demonstrating no predictable patterns. Answer: C 9. Which of the following accurately describes metabolic changes in bone, muscle, and fat that typically occur in the menopausal transition? A. Increases in bone, muscle, and fat mass B. Decreases in bone, muscle, and fat mass C. Increases in muscle and fat mass but a decrease in bone mass D. Decreases in bone and muscle mass but an increase in fat mass Answer: D 10. Which of the following are characteristics of metabolic syndrome? (Select all that apply.) A. Increased lipid levels B. Insulin resistance C. Decreased inflammatory response D. Increased risk of thrombosis E. Increased blood pressure Answer: A, B, D, E 11. The nurse is reviewing assessment and lab results of a patient at risk for metabolic syndrome. Which of the following findings are consistent with metabolic syndrome? (Select all that apply.) A. Waist circumference of 37 inches B. Body mass index (BMI) of 23 C. Blood pressure of 138/86 mmHg D. Fasting blood glucose level of 99 mg/dL E. Triglycerides level of 161 mg/dL Answer: A, C, E 12. Patricia is a 50-year-old woman with a body mass index (BMI) of 31 and elevated lipid levels. Which of the following changes would you recommend to promote Patricia’s health? A. Exercise 60 to 90 minutes per day at moderate intensity, most days B. Limit alcohol intake to no more than two drinks per day C. Take omega-3 fatty acid supplements D. No smoking or use of tobacco E. Reduce waist circumference to less than 40 inches Answer: A, C, D 13. Lorraine is a 46-year-old woman who has a body mass index (BMI) of 21 and who is generally healthy. Based on current recommendations by the National Institutes of Health (NIH), what exercise program would you suggest to Lorraine to maintain her current level of fitness? A. None—she is already maintaining her fitness level B. 20 minutes of light-intensity exercise per day, 3 days per week C. 30 minutes of moderate-intensity exercise per day, 5 days per week D. 60 minutes of heavy-intensity exercise per day, 6 days per week Answer: C 14. Isadora is a 49-year-old woman with a body mass index (BMI) of 18 and a family history of osteoporosis. Which of the following health promotion recommendations specifically related to preventing osteoporosis should the nurse give to Isadora? (Select all that apply.) A. Take aspirin daily B. Include plenty of calcium in the diet C. Consume foods fortified with vitamin D D. Limit sodium intake E. Engage in weight-bearing exercise Answer: B, C, E 15. How much more likely are women to be diagnosed with depression in their lifetime than men? A. 60% B. 70% C. 80% D. 90% Answer: B Chapter 6 Assessing Older Women’s Health Debra Hain 1. How many people in the United States were 65 years or older in 2010? A. 3.1 million B. 20.5 million C. 40.3 million D. 83.7 million Answer: C 2. What percentage of those 85 years and older in the United States in 2010 were women? A. 47% B. 57% C. 67% D. 77% Answer: C 3. What percentage of women 85 years and older in the United States in 2010 were widowed? A. 35% B. 52% C. 73% D. 91% Answer: C 4. Compared with older men, older women are: (Select all that apply.) A. More prepared to retire B. Less likely to retire at retirement age C. More likely to have a retirement pension from their employer D. Less likely to report being satisfied in retirement E. More likely to live alone Answer: B, D, E 5. In 2030, which of the following is the percentage of the older population (65 years or older) that is projected to be White? A. 47% B. 54% C. 67% D. 74% Answer: A 6. Which of the following cognition-related capacities typically decline with normal aging? (Select all that apply.) A. Motivation B. Short- and long-term memory C. Knowledge D. Learning E. Wisdom Answer: A, B, D 7. Daisy, a 77-year-old client, reports a gradual and slight loss of hearing, especially for higher pitched tones. The nurse should recognize this condition as which of the following? A. Presbyopia B. Presbycusis C. Age-related macular degeneration D. Otitis media Answer: B 8. What percentage of cancers are diagnosed at age 55 years or older? A. 55% B. 65% C. 75% D. 85% Answer: C 9. Which of the following is the leading cause of cancer death in women? A. Breast cancer B. Lung cancer C. Colorectal cancer D. Pancreatic cancer Answer: B 10. Mamie is a 76-year-old client who is suspected of having Alzheimer’s disease. To confirm this suspicion, the nurse should look for impairment in which of the following mental functions? (Select all that apply.) A. Visual acuity B. Memory C. Language D. Attention E. Ability to reason Answer: B, C, D, E 11. A phenotype of positive aging that has been described in older women is focused on which of the following? (Select all that apply.) A. Presence of chronic disease B. Physical functioning C. Presence of a disability D. Social functioning E. Emotional functioning Answer: B, D, E 12. Which of the following are the benefits associated with weight-bearing aerobic exercise in older women? (Select all that apply.) A. Decreased risk for breast cancer B. Increased bone density C. Reversal of Alzheimer’s disease symptoms D. Improved balance E. Improved insulin resistance Answer: B, D, E 13. Opal, who is 82 years old, would like information on vaccinations recommended for clients of her age. Which of the following should the nurse recommend? (Select all that apply.) A. Annual influenza B. Human papillomavirus C. Onetime pneumococcal D. Hepatitis B E. Herpes zoster Answer: A, C, E 14. What percentage of community-dwelling older adults take at least five prescription medications? A. 29% B. 39% C. 49% D. 59% Answer: A II Promotion of Wellness for Women Chapter 7 Women and Sexuality Catherine Ingram Fogel, Diane Marie Schadewald 1. Which of the following are dimensions of sexuality? (Select all that apply.) A. Sexual desire B. Sexual identity C. Ethnic identity D. Presentation of self E. Desire for achievement Answer: A, B, D 2. Sexual health requires which of the following? (Select all that apply.) A. A body mass index (BMI) of 18 to 24.9 B. Awareness and acceptance of one’s body C. Sexual intercourse two to three times per week D. The possibility of pleasurable, safe sexual experiences E. The absence of any sexually transmitted infection Answer: B, D, E 3. Which of the following are characteristics of women’s sexual response? (Select all that apply.) A. Highly consistent among a woman’s life stages B. Highly variable among different women C. Expectations and perceptions influenced by culture D. A complex interplay of psychological, physiological, and interpersonal components E. Unaffected by religious beliefs Answer: B, C, D 4. The Masters and Johnson model of human sexual response developed in the 1960s included which series of phases? A. Excitement, plateau, orgasm, resolution B. Desire, excitement, orgasm C. Neutrality, awareness of nonsexual need to be sexual, choice to be stimulated, some sexual arousal, sexual desire, more arousal, physical well-being, spin-offs D. Neutrality, desire, excitement, plateau, orgasm, resolution Answer: A 5. The Kaplan model of human sexual response included which series of phases? A. Excitement, plateau, orgasm, resolution B. Desire, excitement, orgasm C. Neutrality, awareness of nonsexual need to be sexual, choice to be stimulated, some sexual arousal, sexual desire, more arousal, physical well-being, spin-offs D. Neutrality, desire, excitement, plateau, orgasm, resolution Answer: B 6. Basson’s model of female sexual response is based on which of the following premises? A. Women and men experience the same cycle of sexual response. B. Desire is a critical first step in the sexual response cycle. C. Women can enter into sexual activity for reasons other than innate desire. D. Female sexual response follows a linear biologic progression. Answer: C 7. According to a study by Robbins et al., what percentage of 17-year-old females reported ofever having masturbated? A. 28% B. 38% C. 48% D. 58% Answer: D 8. Although there is variation, during what phase of life is it most likely that females, particularly lesbians, will establish their sexual orientation? A. Childhood B. Adolescence C. Adulthood D. Midlife Answer: C 9. The nurse is conducting a sexual assessment on Pamela, a 50-year-old woman. On asking whether Pamela is involved in a sexual relationship, Pamela says “yes.” Which follow-up question would be most appropriate for the nurse to ask next? A. “Are you married?” B. “Are you and your husband satisfied with your sex life?” C. “Who are you having sex with?” D. “Are you involved in a sexual relationship with men, women, or both?” Answer: D 10. The nurse is conducting a sexual assessment with Wendy, a 16-year-old client, who appears nervous. The nurse sees on Wendy’s questionnaire that her period is a week later than usual. What would be the most effective way for the nurse to begin the conversation with Wendy? A. “Many young women your age experience irregular periods for different reasons. Is this something you are experiencing?” B. “So, I see that your period is late. Do you think you are pregnant?” C. “Have you been using condoms when you have sex with your boyfriend?” D. “Don’t worry. It’s highly unlikely that you are pregnant, only being a week late.” Answer: A 11. In the PLISSIT (permission, limited information, specific suggestions, and intensive therapy) model of sexual health care interventions, the first step is which of the following? A. Perform a Pap smear B. Palpate the client’s abdomen C. Ask about the patient’s past medical history D. Ask permission to discuss sexual function Answer: D 12. When conducting a sexual assessment with a client, Ellie, the client gets angry and says that her partner is always so selfish and is only concerned with her own sexual needs. She relates a string of recent experiences with her partner and then says, “What do you think I should do?” Which of the following would be the most appropriate response for the nurse to give? A. “I agree that your partner is being selfish. You might consider breaking up with her.” B. “Why don’t we schedule a follow-up appointment with you and your partner so that we can discuss this together.” C. “When my husband is being selfish, I just refuse to have sex with him. Maybe you should do the same.” D. “Let me refer you to a good sex therapist I know. She is better trained to provide the kind of care you require.” Answer: D 13. A patient reports chronic pain in the vulvar area that has persisted for 4 months. The nurse recognizes this condition as which of the following? A. Vulvodynia B. Vestibulodynia C. Dyspareunia D. Genitopelvic pain/penetration disorder Answer: A 14. A patient reports experiencing fear and anxiety just before having intercourse related to the pain she anticipates experiencing. She also reports feeling uncomfortable tension in her pelvic floor muscles during intercourse. The nurse recognizes this condition as which of the following? A. Vulvodynia B. Vestibulodynia C. Dyspareunia D. Genitopelvic pain/penetration disorder Answer: D 15. A nurse is counseling a 15-year-old client whose boyfriend wants her to try drinking acouple of beers before they have sex. Which of the following effects of alcohol use on sexual function should the nurse mention to the client? A. An intensified orgasm for both her and her boyfriend B. Increased fear and anxiety about having sex C. Increased risk taking in sex, such as not using a condom D. Increased vaginal lubrication Answer: C Chapter 8 H Health Needs of Lesbian, Bisexual, and Transgender Populations 1. The primary cause of health disparities among lesbian, gay, bisexual, and transgender (LGBT) individuals is believed to be which of the following? A. Lack of awareness of need for care B. Increased genetic risk C. Hormonal changes D. Societal stigma and marginalization Answer: D 2. LGBT persons are believed to comprise what percentage of the population? A. 1% to 2.5% B. 3.8% to 10% C. 10.3% to 15.8% D. 16.1% to 21.7% Answer: B 3. A nurse practitioner recently joined a practice with a diverse clientele, including persons of all sexual orientations and gender identities. How should the nurse approach these clients? (Select all that apply.) A. Avoid bringing up sexual orientation or gender identity B. Ask all clients openly about their sexual orientation and gender identity C. Assume all clients are heterosexual and cisgender unless they say otherwise D. Personalize care based on relevant risk factors and individual patient behaviors E. Base treatment plans on risk factors associated with a person’s sexual orientation Answer: B, D 4. What is the prevalence of gender dysphoria in the United States? A. 5% B. 3% C. 1% D. Unknown Answer: D 5. Which of the following is the most accurate statement regarding the etiology of homosexuality? A. It is a genetic trait B. It is determined by hormone levels C. It is likely a combination of nature and nurture D. It is a social construct Answer: C 6. What was the last year that homosexuality appeared as a pathological condition in the Diagnostic and Statistical Manual of Mental Disorders (DSM)? A. 1963 B. 1973 C. 1983 D. 1993 Answer: B 7. Melissa is a 62-year-old lesbian client. On the basis of her age and sexual orientation, Melissa is more likely to do which of the following? (Select all that apply.) A. Live alone B. Have HIV C. Lack family support for meeting basic needs D. Fear discrimination when accessing medical services E. Be homeless Answer: A, C, D 8. What condition related to gender identity is included in the Diagnostic and Statistical Manual of Mental Disorders, fifth edition (DSM-5)? A. Gender dysphoria B. Gender identity disorder C. Transgender disorder D. Body dysmorphic disorder Answer: A 9. Jackie is a 19-year-old transwoman who has arrived at the office today for a physical. What health disparities should the nurse assess for in Jackie? (Select all that apply.) A. Risky sexual behaviors B. Depression C. Multiple sclerosis D. Trauma and violence E. Substance abuse Answer: A, B, D, E 10. Meg is a 15-year-old bisexual client. Which of the following is Meg at a greater risk for than her heterosexual counterparts? (Select all that apply.) A. Spinal cord injury B. Experiencing school bullying C. Bipolar disorder D. Homelessness E. Suicidality Answer: B, D, E Chapter 9 Health Needs of Women with Disabilities Michele Davidson 1. The inability to carry out relevant roles due to the social impact of either a physical impairment or a functional limitation is known as which of the following? A. Disorder B. Disease C. Disability D. Distress Answer: C 2. Which of the following is a condition that is “age related” rather than “ageing related”? A. Cardiomyopathy B. Osteoarthritis C. Presbyopia D. Presbycusis Answer: A 3. Which of the following is a condition that is “ageing related” rather than “age related”? A. Presbyopia B. Breast cancer C. Atherosclerosis D. Chronic obstructive pulmonary disease Answer: A 4. Which of the following are mechanisms by which early-onset injuries and illnesses increase the risk of loss of benefits, income, and health? (Select all that apply.) A. Eligibility for government-sponsored assistance programs B. Loss of work incentive C. Loss of the ability to work D. Development of empathy for others with disabilities E. Exacerbation of preexisting conditions Answer: B, C, E 5. Sadie is a 52-year-old client with chronic back pain caused by an injury at work that has become a permanent disability. Which resources should the nurse help Sadie access benefits from? (Select all that apply.) A. Veterans Administration B. Workers’ compensation C. Women, Infants, and Children D. Social Security Disability Insurance E. Supplemental Nutrition Assistance Program Answer: B, D 6. Which of the following is a risk factor for declining health in a woman with a later onset disability? A. Age of 40 years or older B. Age of 50 years or older C. Age of 60 years or older D. Age of 70 years or older Answer: C 7. Which of the following is a risk factor for declining health in a woman with a disability? A. Early onset of menopause B. Late onset of menopause C. Early onset of menarche D. Late onset of menarche Answer: A 8. Which of the following is a risk factor for declining health in a woman with a disability? A. Having only seven teeth left B. Having had a root canal early in life C. Having a dental implant D. Wearing dentures Answer: A 9. To avoid a decline in health related to a sedentary lifestyle, the nurse should encourage Doris, a 63-year-old woman who is blind, to engage in planned physical activity at least for how many days per week? A. 1 B. 3 C. 5 D. 7 Answer: B 10. Which condition is a risk factor for declining health in a woman with a disability? A. Dementia B. Dyslexia C. Attention deficit hyperactivity disorder D. Obsessive-compulsive disorder Answer: A Chapter 10 Integrating Wellness: Complementary Therapies and Women’s Health J o Lynne W. Robins MULTIPLE CHOICE 1. A pregnant woman tells the nurse that she got relief from nausea when she had a therapy that involves pressure and massage on meridian sites. What type of therapy does this describe? a. Acupuncture b. Acupressure c. Aromatherapy d. Ayurveda ANS: B Acupressure uses finger pressure and massage on the meridian sites. It can be used during pregnancy to control nausea, backache, and pain. It has been useful for minor postpartum problems such as constipation. 2. Which child should not receive massage therapy? a. 15-year-old with a fractured femur b. 12-year-old with diabetes mellitus c. 8-year-old with Down syndrome d. 17-year-old with an eating disorder ANS: C Children with Down syndrome are prone particularly to cervical spine anomalies and may be injured by massage therapy. 3. A 12-year-old with rheumatoid arthritis finds aromatherapy helpful for relieving her joint discomfort. Which essential oil is useful for children with chronic pain? a. Lavender b. Ephedra c. Ginseng d. Kava-kava ANS: A Lavender, chamomile, and sandalwood essential oils are useful in aromatherapy for children with chronic pain. 4. A pregnant woman wishes to use aromatherapy during her labor and delivery. What is the most appropriate essential oil for the nurse to recommend? a. Juniper b. Wintergreen c. Thyme d. Citrus ANS: D Citrus is one essential oil that has been shown to be useful during labor and delivery. 5. A parent asks the nurse, What is guided imagery? Which statement is the most accurate response? a. It is a technique where the patient focuses on an image to relieve stress. b. It involves using water to promote relaxation. c. The patient enters a hypnotic state of sleep to promote relaxation. d. It helps the patient recognize tension in the muscles with responses on an electronic machine. ANS: A In guided imagery, by focusing on a specific image, stress reduction and improved performance can result. 6. A woman taking St. Johns wort and ginseng daily is scheduled to have a hysterectomy in 3 weeks. What instruction should the nurse provide? a. The herbs are not likely to cause any problems during the surgery. b. The St. Johns wort must be stopped prior to surgery, but she can continue the ginseng. c. The ginseng should be stopped 1 week before surgery. d. She should discontinue taking both herbs 2 weeks before surgery. ANS: D Both St. Johns wort and ginseng can cause problems during surgery, and their use should be discontinued 2 weeks before surgery. 7. Which herb can the nurse suggest to be used for discomforts associated with menopause, such as hot flashes? a. Evening primrose oil b. Echinacea c. Milk thistle d. Black cohosh ANS: D Black cohosh diminishes hot flashes by reducing luteinizing hormone. It also reduces joint pain and other menopausal discomforts. 8. A young mother asks, Is there an alternative medicine for children with asthma? Which form of alternative medicine would be the most helpful for the nurse to suggest? a. Reflexology b. Rolfing c. Guided imagery d. Acupressure ANS: C The use of guided imagery has helped relieve some of the symptoms of asthma. 9. What is the difference between complementary therapy and alternative therapy? a. Complementary therapy must be administered by a medical doctor. b. Complementary therapy is administered with conventional therapy. c. Complementary therapy replaces conventional therapy. d. Complementary therapy is administered to a group of patients at the same time. ANS: B Complementary therapy is administered with conventional therapy, such as massage with muscle relaxants for low back pain. 10. The nurse uses a diagram to show the location of meridians. How will the nurse explain the definition of meridians? a. They are lymph nodes. b. They are invisible pathways for energy. c. They are lines that divide the body into 10 zones. d. They are areas of skin that are specifically innervated. ANS: B Meridians are invisible pathways through which energy travels to effect acupuncture treatment. 11. Which herbal remedy used by a patient taking warfarin should the nurse report to the physician? a. Angelica (dong quai) b. Chamomile c. Ginseng d. Kava-kava ANS: A Angelica prolongs prothrombin time and will synergize the effect of the warfarin. 12. What should the nurse remind a parent who is considering homeopathic remedies for treatment of her childs asthma? a. Should be drunk with large amounts of fluid b. Can be taken with traditional Western medications c. Can be enhanced by drinking hot tea d. May contain mercury, alcohol, or arsenic ANS: D Homeopathic remedies often contain mercury, alcohol, or arsenic and are taken sublingually. All Western medications should be stopped when the homeopathic therapy is begun. Caffeine drinks are to be avoided during homeopathic treatment. 13. The focus of acupressure is to restore the balance of what? a. Chi b. Shiatsu c. Yin and yang d. Ayurveda ANS: A Acupressure is focused on the return of the balance of Chi to control disease processes. 14. A breastfeeding mother tells the nurse she is taking large doses of vitamin C to keep up her energy. What should the nurse warn that large doses of vitamin C can cause in an infant? a. Diarrhea b. Jaundice c. Colic d. Retinal damage ANS: C Vitamin C can be passed on to a breastfeeding child through breast milk and can cause colic. 15. The pregnant patient with a stasis ulcer asks if she might be a candidate for hyperbaric oxygen therapy (HBOT). What is the nurses best response? a. Yes. Hyperbaric oxygen therapy should have no harmful effect on your baby. b. No. High amounts of oxygen in your system will cause changes in your babys heart. c. Yes. Hyperbaric oxygen therapy is a much better option than using antibiotics. d. No. Hyperbaric oxygen therapy may cause the placenta to separate fromthe uterine wall. ANS: B High concentrations of oxygen in the mothers blood can cause closure of the ductus arteriosus and cause fetal death. 16. A patient is providing history information to the admitting nurse about treatment used for chronic pain. The patient reports she participates in a type of relaxation therapy that enables her to recognize tension in the muscles via responses on an electronic machine and visual electromyography responses. What type of therapy does the nurse record on admission record? a. Guided imagery b. Biofeedback c. Hypnotherapy d. Chiropractic care ANS: B Biofeedback is a type of relaxation therapy that enables the patient to recognize tension in the muscles via responses on an electronic machine and visual electromyography responses. The process is also used by traditional health care providers for drug addiction and chronic pain control. MULTIPLE RESPONSE 17. What conditions would a nurse expect to see treated with hyperbaric oxygen therapy (HBOT)? (Select all that apply.) a. Wounds b. Carbon monoxide poisoning c. Hyperemesis gravidarum d. Decompression illness e. Pneumonia ANS: A, B, D Hyperbaric oxygen therapy (HBOT) uses an airtight enclosure to provide compressed air or oxygen under increased pressure. HBOT is used to revive children with carbon monoxide poisoning, to aid wound healing, and to treat the diving syndrome known as decompression illness. HBOT is contraindicated during pregnancy, because the increased oxygen saturation can cause the ductus arteriosus to close, resulting in fetal death. 18. The mother of a pediatric patient asks the nurse about safety concerns with using herbal supplements with children. Which herbal products would the nurse educate this mother are safe to use in most of the pediatric population? (Select all that apply.) a. Ephedra b. Ginger c. Fish oil d. Chamomile e. Aloe vera ANS: B, C, D, E Ginger, fish oil, chamomile and aloe vera are safe herbal products for children. However, some herbs, such as ephedra, can be fatal to children. 19. The nurse points out that light therapy is used in the treatment of patients with which disorder(s)? (Select all that apply.) a. Digestive disorders b. Seasonal affective disorder c. Inflammatory diseases d. Stress disorders e. Jaundice ANS: B, E Light therapy has proven effective in the treatment of persons with seasonal affective disorders. Light therapy is also used in the treatment of jaundiced babies. 20. What advantage(s) of alternative health care should the nurse outline when providing information to patients? (Select all that apply.) a. Offering more patient control of health care b. Offering a variety of health care advisors c. Keeping patients from having to make decisions d. Using natural products rather than chemical ones e. Incorporating cultural beliefs and practices ANS: A, B, D, E Alternative health care actually promotes the patients decision making in care. 21. Which approaches to care are combined with osteopathy? (Select all that apply.) a. Manipulation therapy b. Aroma therapy c. Herbal application d. Pressure point therapy e. Traditional medicine ANS: A, D, E DIF: III Promotion of Gynecologic Health Care Chapter 11 Menstruation and Related Problems and Concerns Kristine Alswager, Christine Durler, Deborah Griswold 1: The symptoms of premenstrual syndrome typically resolve a: by the end of menses b: about 2 weeks after the start of menses c: 2 weeks before the onset of menses d: at the time of ovulation 2: In premenstrual syndrome, there is a symptom-free interval of at least a: 1 week per cycle b: 1 cycle out of 4 c: 2 weeks per cycle d: 2 cycles per year 3: The maximum estimated incidence of women with some physical or emotional premenstrual symptoms is a: 60%-70% b: 45%-55% c: 30%-40% d: 75%-85% 4: Premenstrual dysphoric disorder occurs in what percent of women? a: 5% b: 75% c: 50% d: 25% 5: In making the diagnosis of premenstrual syndrome or PMDD, the characteristic that is most distinguishing is the a: duration of symptoms b: cyclic occurrence of symptoms c: degree of disability in the patient d: severity of symptoms 6: Which of the following is most likely related to the underlying etiology of PMDD? a: Serotoni n b: Catecholamine s c: Norepinephrin e d: Progesterone 7: The most useful diagnostic tool with respect to premenstrual syndrome is a: a careful history b: serial progesterone levels c: a prospective menstrual diary d: serial blood glucose determinations 8: Reduction or elimination of which of the following dietary components appears to have a positive effect in PMS patients? a: Calciu m b: Fa t c: Fresh fruit d: Magnesium 9: Which of the following is most likely to help a patient's PMS symptoms a: Vaginal progesterone b: Transdermal estrogen c: Oral medroxyprogesterone acetate d: Combined oral contraceptives 10: A 25-year-old woman with no medical problems comes in for her annual exam. She expresses the worry that she has PMDD. She reports feeling bloated and moody when she has her period. The most appropriate initial management for this patient is a: treating her for hypothyroidism b: asking her to keep a menstrual diary for 2 months c: ordering a pelvic ultrasound d: giving her a prescription of fluoxetine to be taken only during the luteal phase 11: A patient keeps a menstrual diary for 2 months. The diary reveals that she has bloating, fatigue, anxiety, insomnia, and tearfulness in the days prior to her menses and that the symptoms resolve completely after her menses starts. She reports that her insomnia is so bad that it prevents her from being efficient at work because she is not well rested. The most likely diagnosis for this patient is a: bipolar disorder b: PMDD c: major depressive disorder d: PMS 12: A patient with PMDD asks for some treatment options. You respond that the treatment that has clearly demonstrated benefit for her condition is a: GnRH agonist b: sertralin e c: danazo l d: ovarian suppression with high-dose depot medroxyprogesterone 13: Which of the following has been shown to significantly improve the symptoms of women with PMS? a: Light therapy b: Vitamin E c: Evening primrose d: Aerobic exercise 14: Which of the following would be the first line of treatment in the management of symptoms of PMS? a: Selective serotonin reuptake inhibitors b: GnRH agonist c: Complex carbohydrate diet d: Oral contraceptive 15: An important diagnosis to distinguish from PMDD is major depressive disorder. Which of the following is increased in patients with major depressive disorder but not in patients with PMDD? a: Sleep disturbance b: Suicide risk c: Anxiet y d: Disordered eating [ANS: a,a,d,a,b,a,c,b,d,b,b,b,d,c,b] Chapter 12 Contraceptive Management and Family Planning Beth Walcker, Coralie Pederson, Kathryn A. Caufield A 20-year-old white female G0 comes to see you about contraception. She is in college and plans to go to grad school, so she wants something that is 100% effective in preventing pregnancy. You explain to her that the method failure rate and the typical use rate can be very different depending on the method chosen. 1: A. The "method failure rate" reflects the rate of failure when the method is a: compared with the use of no contraception at all b: used correctly 100% of the time c: used by a random sample of women d: used in laboratory animals e: used in the laboratory setting 2: B. The most common method of contraception among younger women in the United States is a: long-acting hormone (rod or injection) b: oral contraceptive c: rhyth m d: intrauterine device (IUD) e: spermicidal foam 3: C. The pregnancy rate (% pregnant in 1 year) for a couple using no method ofcontraception typically is a: 25 b: 65 c: 15 d: 45 e: 85 4: D. In the first year of use, the typical use pregnancy rate for a woman using a diagphragm and spermicide a: 1 6 b: 2 0 c: 2 8 d: 8 e: 24 5: E. In the first year of use, the typical use pregnancy rate for a woman using a copper intrauterine contraceptive is a: 8 b: 1.5 c: 0.8 d: 10 e: 3 6: F. Which of the following is the least effective birth control option? a: Combination oral contraceptives b: Cervical cap c: Monthly transvaginal ring d: Weekly transdermal patch e: Three-month injectable 7: G. The failure of oral contraceptives usually is related to a: interference by other medications that the patient is taking b: an inherent problem in the estrogen-to-progesterone ratio c: altered gastrointestinal absorption of the oral contraceptive caused by hormonal influences d: ingestion of the oral contraceptive agent with alcohol e: missed doses of the oral contraceptive 8: H. Which of the following contraceptive techniques relies the least on patient compliance? a: Contraceptive ring b: Combination pill c: Progestin pill d: Contraceptive patch e: Contraceptive implant A 24-year-old Latin American female G2P2 sees you for her annual. She is about to wean her baby in a few months and is on the progesterone-only pill. She really likes this particular pill because she has had no periods while breastfeeding. She wonders if she can continue to use it. 9: A. You counsel her that one of the main reasons to consider switching to combination birth control pills is a: better lipid profile b: better control of her moods c: better cycle control d: better weight loss e: better control of her acne 10: B. Which of the following statements about the progestin-only minipill is correct? a: It cannot be started postpartum until at least 2 months after delivery b: It is taken once per week c: It is most useful for women under 30 years d: It is most useful for lactating women A 34-year-old female G4P4 sees you for her annual exam. You notice that she uses natural family planning for contraception. She states she desires no more pregnancies. You discuss other options including long-acting reversible contraception and permanent sterilization. She declines for religious reasons. 11: A. Your next step should be to a: refuse to continue to see her as a patient b: laughingly tell her she is crazy and will probably see her pregnant again next year c: give her information on fertility awareness and encourage her to contact you with any questions d: advise her to talk to her partner and come back to discuss the options e: remind her that she is essentially using luck for contraception 12: B. This woman's periods are regular, coming every 28 days. This patient's "fertile" period would be days a: 10-17 b: 7-14 c: 7-17 d: 10-20 13: C. Which of the following methods is most appropriate for this woman requesting a "natural" method of family planning? a: Cervical cap b: Cervical mucus method c: Vaginal ring d: Female condom A 20-year-old college student G0 sees you for her annual. Her gyn history is remarkable for menarche at 13 years and regular cycles, but her periods last 7 to 9 days and are very painful. Her college roommate had similar periods and got good results with birth control pills. During the history you note that she smokes 1 pack per day. She has migraines that are preceded by flashing lights. Her mother had breast cancer diagnosed at age 49 years and both her parents are hypertensive. 14: A. You are hesitant to prescribe birth control pills because of a: her migrines preceded by flashing lights b: her painful periods that may be consistent with endometriosis c: her smoking one pack per day d: her history of breast cancer in her mother e: her family history of hypertension 15: B. What is the primary mechanism of action of oralcontraceptives? a: Inhibit ovulation b: Impose a barrier between the sperm and the egg c: Create an environment in the uterus that is toxic to the sperm d: Destroy the embryo shortly after fertilization e: Alter the ability of the fertilized egg to implant and grow 16: C. Which is a contraindication to the birth control pill? a: Smoking in a woman older than 35 years b: A family history of stomach cancer c: Dysmenorrhe a d: A history of ovarian cysts 17: D. Other than investigating the cause, what action should be taken when a woman using anoral contraceptive experiences a headache? a: Increase oral contraceptive to twice daily b: Switch to the phasic pill c: Continue oral contraceptive d: Discontinue all contraception e: Discontinue oral contraceptive A 16-year-old sees you in the Derm Clinic. She is hesitant because she heard antibiotics made "your birth control not work." You reassure her that most common antibiotics do not lessen the effectiveness of birth control pills. 18: A. When taken concurrently, which of the following will reduce the effecacy of oral contraceptives? a: Tetracyclin e b: Aspiri n c: Phenytoi n d: Metronidazol e e: Methyldopa 19: B. Which of the following best describes "phasic" formulation of oralcontraceptives? a: They contain only progestin in varying doses b: They contain varying doses of hormones throughout the cycle pack c: They are mostly used postpartum d: They contain a higher dose of hormones per month than monophasic contraceptive pills e: They have a different mechanism of action than monophasic pills 20: In which of the following casses should biphasic oral contraceptives be discontintued? A: Galactorrhea b:Right upper quadrant pain c:Amenorrhea d:Anemia e:Hepatic mass with tenderness ANS: [b,b,e,c,c,b,e,e,c,d,c,a,b,a,a,a,c,c,b,e] Chapter 13 Infertility Michelle Valentine, Jennifer R. Gardella 1. Ingrid is a 32-year-old client who has struggled to get pregnant and has come to the office today to find out if she might be infertile. In Ingrid’s case, which of the following criteria must be met for infertility to be diagnosed, as should be explained by the nurse? (Select all that apply.) A. No prior pregnancies B. Regular sexual intercourse C. Unprotected sexual intercourse D. Well-timed sexual intercourse E. At least 12 months of actively attempting to get pregnant Answer: B, C, D, E 2. At what age is it estimated that fertility begins to decline in women? A. 30 B. 35 C. 40 D. 45 Answer: B 3. For the period 2011 to 2013, about what percentage of women aged 15 to 44 years reported difficulty getting pregnant or carrying a baby to term? A. 24% B. 34% C. 44% D. 54% Answer: A 4. Bonnie is a 31-year-old client who has been trying to get pregnant for 18 months now. After interviewing Bonnie and going over her health record, which of the following are the risk factors for infertility, as should be mentioned by the nurse to Bonnie? (Select all that apply.) A. Her age (31 years) B. Drinking two beers per day C. Body mass index (BMI) of 16 D. Strict vegetarian diet E. Walking a mile each day for exercise Answer: B, C, D 5. Minnie is a 21-year-old client who has just been diagnosed with chlamydia. What complication associated with this infection that can lead to infertility should the nurse inform Minnie of? A. Pelvic inflammatory disease B. Epididymitis C. Ovarian cancer D. Endometriosis Answer: A 6. Mike, a 40-year-old client, is coming to the office to undergo assessment related to infertility. Which of the following should the nurse tell Mike to be prepared to do? A. Undergo a digital rectal exam B. Have a urinary catheter placed C. Undergo an EKG D. Masturbate to provide a semen specimen Answer: D 7. Which of the following are risk factors for males related to infertility? (Select all that apply.) A. Undescended testicles B. History of radiography of the groin C. Onset of puberty at the age of 13 years D. Erectile dysfunction E. Mumps-related orchitis Answer: A, B, D, E 8. Hope, a 35-year-old client, has been diagnosed with oligomenorrhea. Which of the following are medications the nurse should consider prescribing to Hope to induce ovulation and aid conception? (Select all that apply.) A. Mifepristone B. Levonorgestrel C. Clomiphene citrate D. Aromatase inhibitor E. Human chorionic gonadotropin Answer: C, D, E 9. Which assistive reproductive technology has a higher fecundability than natural conception? A. Intrauterine insemination B. Gamete intrafallopian transfer C. In vitro fertilization D. Zygote intrafallopian transfer Answer: C 10. What recommendations related to use of reproductive technologies should the nurse follow to help avoid ethical problems? (Select all that apply.) A. Progress from invasive to conservative measures B. Avoid excessive treatments and costs C. Provide full disclosure regarding success rates of a procedure before initiating it D. Offer counseling for gamete donors, surrogates, and their partners E. Insist that the offspring conceived by gamete donation be informed ofthe nature of conception by the age of 12 years Answer: B, C, D 11. A client with ovarian cancer who will be undergoing surgery and radiation therapy is concerned that she will become infertile as a result of the treatment. Which of the following should the nurse recommend that the client consider? A. Cryopreservation of her oocytes B. Estrogen therapy C. Intrauterine insemination D. Adoption Answer: A Chapter 14 Vaginitis and Sexually Transmitted Diseases Susan D. Schaffer, Jane Houston A 16-year-old virginal is brought in by her mother because of possible vaginal infections. Which of the following descriptions is most consistent with normal (physiologic) discharge? a: White, clear, and flocculent b: White, curds, cottage cheese- like c: Gray-green, thin, and adherent d: Gray-green, frothy 2: A 24-year-old patient requests treatment for what she believes is a bacterial infection of the vagina. Which of the following is characteristic of vaginal discharge in cases of bacterial vaginosis? a: Fishy odor b: pH<4.5 c: Whitish color d: Curdy consistency 3: A 40-year-old patient has recently been treated for a urinary tract infection with antibiotics. She now complains of a possible yeast infection. Discharge in cases of vaginal candidiasis is characterized by a: presence of hyphae and buds b: thin, homogeneous consistency c: white cells d: foul odor 4: A 30-year-old patient is seen for a vaginal discharge that started after she had relations with a new partner. Which of the following characterizes vaginal trichomoniasis? a: pH of discharge less than 4.5 b: Vaginal dryness c: White discharge d: Motile protozoa on microscopic examination 5: A 55-year-old patient has recently developed vaginal irritation. Her last menstrual period was 9 months ago. Which of the following is characteristic of atrophic vaginitis? a: Increased circulating estrogen levels b: Thickened vaginal epithelium c: Vaginal dryness d: pH of vagina less than 4.5 6: Most of the liquid portion of physiologic vaginal secretions in a woman of reproductive age comes from a: vaginal transudate b: the cervix c: the Skene glands d: the Bartholin glands 7: In a 33-year-old woman with regular periods, the creamy white portion of normal vaginal secretions comes from a: exfoliated squamous epithelial cells b: vaginal white blood cells c: cervical mucus d: the Bartholin glands 8: A 19-year-old patient who was concerned about vaginal discharge was found to have normal vaginal secretions. Which of the following is the most appropriate management? a: Return as needed. b: Return after a single pill of fluconazole. c: Return after a 3-day course of metronidazole. d: Return after the next period. 9: The normal pH of vaginal secretions in a reproductive-age woman is a: 3.5-4.5 b: 6.5-7.5 c: 5.0-6.0 d: 8.0-9.0 10: What is the difference in the normal pH of the vagina before and after menopause? a: Same in pre- and postmenopausal patient b: Varies depending on the phase of the premenopausal cycle c: Higher in premenopausal patient d: Higher in postmenopausal patient 11: A 38-year-old patient requests treatment for a yeast infection but declines an examination because she is on her period. The most common symptom associated with vulvovaginal candidiasis is a: pai n b: feve r c: fishy smelling discharge d: pruritus 12: A 38-year-old patient requests evaluation of a fishy-smelling discharge. Which of the following is a criterion for the diagnosis of bacterial vaginosis? a: Presence of clue cells b: Negative "whiff" test c: Cottage cheese-like discharge d: pH less than 4.5 13: A 45-year-old woman complains of a copious vaginal discharge. Which of the following is a common cause of increased vaginal discharge? a: Trichomonias is b: Herpe s c: Human papillomavirus d: Atrophic vaginitis 14: A 25-year-old patient complains of vaginal discharge that is white and curdy in appearance. What is the appropriate method to establish the diagnosis? a: Constellation of symptoms b: Odor of the discharge c: The amount of discharge d: Microscopic examination of the discharge 15: What is the normal ratio of anaerobic to aerobic bacteria in the vagina? a: 1 to 5 b: 5 to 1 c: 1 to 1 d: 10 to 1 16: A 21-year-old patient complains of a vaginal discharge but is also concerned about pelvic inflammatory disease (PID). Which type of vaginitis is associated with PID? a: Trichomonas vulvovaginitis b: Atrophic vaginitis c: Desquamative inflammatory vaginitis d: Vaginal candidiasis 17: Which of the following best describes a "clue cell"? a: Clumped white blood cells b: Vaginal epithelial cells with adherent bacteria c: Keratinized vaginal epithelial cells with adherent white blood cells d: Immature vaginal epithelial cells 18: A 30-year-old patient is found to have Trichomonas. She asks about multiple other symptoms that she is concerned about. She can be informed that Trichomonas vaginalis is a flagellate protozoan that can live in the a: oropharyn x b: vagin a c: bladde r d: anus 19: In which of the following is treatment of sexual partners recommended? a: Trichomonas vaginitis b: Atrophic vaginitis c: Bacterial vaginosis d: Vulvovaginal candidiasis 20: A 29-year-old patient is undergoing evaluation for a symptomatic discharge. Which of the following characterizes the microscopic appearance of Trichomonas? a: Ground-glass cytoplasm b: Pseudohypha e c: Flagellate protozoan d: Bacillus 21: What characteristic lesions may be found in the upper vagina or on the cervix of patients with Trichomonas vaginalis? a: Ulcer s b: White plaques c: Clear blisters d: Petechiae 22: A 26-year-old patient is told by her boyfriend that he has Trichomonas. She has developed a copious discharge recently. The standard treatment for Trichomonas vaginal infection is a: oral clindamycin b: oral fluconazole c: oral metronidazole d: topical synthetic imidazoles 23: A 30-year-old patient is receiving a prescription for metronidazole for the treatment of Trichomonas. What additional advice should she receive? a: Avoid nonsteroidal anti-inflammatory agents b: Avoid alcohol c: Avoid caffeine d: Take vitamin supplementation 24: A 22-year-old patient inquires as the origin of the yeast infection for which she is being treated. She can be informed that the most common source of monilial infections of the vagina is a: sexual contact with an infected partner b: bath water retained in the vagina following bathing c: contaminated clothing d: airborne colonization 25: Roughly 90% of vaginal "yeast" infections are caused by a: Candida tropicalis b: Torulopsis glabrata c: Candida glabrata d: Candida albicans ANS: [aaadc baaad daadb abbac dcbdd] Chapter 15 Women and HIV Dina Treloar, Susan D. Schaffer The nurse is developing a plan of care for a client who is receiving highly active antiretroviral therapy (HAART) for treatment of HIV. The goal of this therapy is to: A) Promote the progression of disease B) Intervene in late-stage AIDS C) Improve survival rates D) Conduct additional drug research A woman who is HIV-positive is receiving HAART and is having difficulty with compliance. To promote adherence, which of the following areas would be most important to assess initially? A) The womans beliefs and education B) The womans financial situation and insurance C) The womans activity level and nutrition D) The womans family and living arrangements When developing a teaching plan for a community group about HIV infection, which group would the nurse identify as an emerging risk group for HIV infection? A) Native Americans B) Heterosexual women C) New health care workers D) Asian immigrants After teaching a group of adolescents about HIV, the nurse asks them to identify the major 4. means by which adolescents are exposed to the virus. The nurse determines that the teaching was successful when the group identifies which of the following? A) Sexual intercourse B) Sharing needles for IV drug use C) Perinatal transmission D) Blood transfusion The nurse reviews the CD4 cell count of a client who is HIV-positive. A result less than which of the following would indicate to the nurse that the client has AIDS? A) 1,000 cells/mm3 B) 700 cells/mm3 C) 450 cells/mm3 D) 200 cells/mm3 When obtaining the health history from a client, which factor would lead the nurse to suspect that the client has an increased risk for sexually transmitted infections (STIs)? A) Hive-like rash for the past 2 days B) Five different sexual partners C) Weight gain of 5 lbs in 1 year D) Clear vaginal discharge Assessment of a female client reveals a thick, white vaginaldischarge. She also reports intense itching and dyspareunia. Based on these findings, the nurse would suspect that the client has: A) Trichomoniasis B) Bacterial vaginosis C) Candidiasis D) Genital herpes simplex A client with trichomoniasis is to receive metronidazole (Flagyl). The nurse instructs the client to avoid which of the following while taking this drug? A) Alcohol B) Nicotine C) Chocolate D) Caffeine A woman gives birth to a healthy newborn. As part of the newborns care, the nurse instills erythromycin ophthalmic ointment as a preventive measure related to which STI? A) Genital herpes B) Hepatitis B C) Syphilis D)Gonorrhea 10.Which findings would the nurse expect to find in a client with bacterial vaginosis? A) Vaginal pH of 3 B) Fish-like odor of discharge C) Yellowish-green discharge D) Cervical bleeding on contact A pregnant woman diagnosed with syphilis comes to the clinic for a visit. The nurse discusses 11.the risk of transmitting the infection to her newborn, explaining that this infection is transmitted to the newborn through the: A) Amniotic fluid B) Placenta C) Birth canal D) Breast milk 12.The nurse encourages a female client with human papillomavirus (HPV) to receive continued follow-up care because she is at risk for: A) Infertility B) Dyspareunia C) Cervical cancer D) Dysmenorrhea 13.A client is diagnosed with pelvic inflammatory disease (PID). When reviewing the clients medical record, which of the following would the nurse expect to find? (Select all that apply.) A) Oral temperature of 100.4 degrees F B) Dysmenorrhea C) Dysuria D) Lower abdominal tenderness E) Discomfort with cervical motion F) Multiparity 14.Which instructions would the nurse include when teaching a woman with pediculosis pubis? A) Take the antibiotic until you feel better. B) Wash your bed linens in bleach and cold water. C) Your partner doesnt need treatment at this time. D) Remove the nits with a fine-toothed comb. 15.A client with genital herpes simplex infection asks the nurse, Will I ever be cured of this infection? Which response by the nurse would be most appropriate? A) There is a new vaccine available that prevents the infection from returning. B) All you need is a dose of penicillin and the infection will be gone. C) There is no cure, but drug therapy helps to reduce symptoms and recurrences. D) Once you have the infection, you develop an immunity to it. A nurse is preparing a presentation for a group of women at the clinic who have been diagnosed 16.with genital herpes. Which of the following would the nurse expect to include as a possible precipitating factor for a recurrent outbreak? (Select all that apply.) A) Exposure to ultraviolet light B) Exercise C) Use of corticosteroids D) Emotional stress E) Sexual intercourse. 17.After teaching a class on sexually transmitted infections, the instructor determines that the teaching was successful when the class identifies which statement as true? A) STIs can affect anyone if exposed to the infectious organism. B) STIs have been addressed more on a global scale. C) Clients readily view the diagnosis of STI openly. D) Most individuals with STIs are over the age of 30. A group of students are reviewing information about STIs. The students demonstrate 18.understanding of the information when they identify which of the following as the most common bacterial STI in the United States? A) Gonorrhea B) Chlamydia C) Syphilis D) Candidiasis 19.A nurse is assessing a client for possible risk factors for chlamydia and gonorrhea. Which of the following would the nurse identify? A) Asian American ethnicity B) Age under 25 years C) Married D) Consistent use of barrier contraception 20.A nurse at a local community clinic is developing a program to address STI prevention. Which of the following would the nurse least likely include in the program? A) Outlining safer sexual behavior B) Recommending screening for symptomatic individuals C) Promoting the use of barrier contraceptives D) Offering education about STI transmission After teaching a class on preventing pelvic inflammatory disease, the instructor determines that 21.the teaching was successful when the class identifies which of the following as an effective method? A) Advising sexually active females to use hormonal contraception B) Encouraging vaginal douching on a weekly basis. C) Emphasizing the need for infected sexual partners to receive treatment D) Promoting routine treatment for asymptomatic females as risk 22.A group of nursing students are reviewing information about vaccines used to prevent STIs. The students would expect to find information about which of the following? A) HIV B) HSV C) HPV D) HAV E) HBV A mother brings her 12-year-old daughter in for well-visit checkup. During the visit, the nurse 23.is discussing the use of prophylactic HPV vaccine for the daughter. The mother agrees and the daughter receives her first dose. The nurse schedules the daughter for the next dose, which would be given at which time? A) In 2 month B) In 2 months C) In 3 months D) In 4 months 24.A woman comes to the clinic complaining of a vaginal discharge. The nurse suspects trichomoniasis based on which of the following? (Select all that apply.) A) Urinary frequency B) Yellow/green discharge C) Joint pain D) Blister-like lesions E) Muscle aches 25.A nurse is teaching a women with genital ulcers how to care for them. Which statement by the client indicates a need for additional teaching? A) I need to wash my hands after touching any of the ulcers. B) I need to abstain from intercourse primarily when the lesions are present. C) I should avoid applying ice or heat to my genital area. D) I can try lukewarm sitz baths to help ease the discomfort. Answer Key 1. C 2. A 3. B 4. A 5. D 6. B 7. C 8. A 9. D 10.B 11.B 12.C 13.B, C, D, E 14.D 15.C 16.A, C, D, E 17.A 18.B 19.B 20.B 21.C 22.C, D, E 23.B 24.A, B 25.B Chapter 16 Common Gynecologic Pelvic Disorders Jennifer M. Laubach, Reena P. Lorntson, Donna E. Forrest 1: Enterocele is defined as herniation of what organ through the vagina? a: Uteru s b: Bladde r c: Small bowel d: Urethra 2: A 45-year-old patient complains of frequent loss of urine when she coughs, laughs, or strains. The volume lost is small. She does not report any dysuria. Which type of incontinence does she have? a: Urg e b: Overflo w c: Stres s d: Psychogenic 3: A 45-year-old patient complains of frequent loss of urine. The volume lost is small, but it occurs almost continuously. She denies urgency, has no dysuria, and voids frequently but in small amounts. She does not ever feel "full" but also never has the sense that she has completely emptied her bladder. What type of incontinence best describes her condition? a: Behavior al b: Overflo w c: Urg e d: Stress 4: A 22-year-old patient complains of occasional loss of urine. She reports a sense of intense fullness and "a need to go" just before the urine is lost. She voids infrequently but in large amounts. She does not ever feel that she "gets enough warning" to get to the bathroom. What type of incontinence does she have? a: Overflo w b: Psychogeni c c: Urg e d: Stress 5: A 22-year-old patient complains of continuous loss of urine since her cesarean delivery 1 month ago. This patient's problem is most likely due to a: vesicovaginal fistula b: stress incontinence c: overflow incontinence d: behavioral incontinence 6: A cystocele may best be demonstrated clinically by a: use of a Sims speculum to retract the anterior vaginal wall b: gentle traction on the cervix c: observing posterior rotation of the anterior vaginal wall in response to change in position d: use of a Valsalva maneuver 7: A patient who loses urine when she coughs or sneezes most likely has a(n) a: urethral prolapse (urethrocele) b: enterocele c: rectocel e d: cystocele 8: A "Q-tip test" is used to evaluate a: residual urine b: urethral mobility c: posterior vaginal wall support d: uterine prolapse 9: When performing a "Q-tip test," incontinence is generally associated with upward rotation of a: 45 degrees b: 10 degrees c: 20 degrees d: 30 degrees 10: What term is applied when the cervix descends below the vulva? a: Rectocel e b: Enterocel e c: Procidenti a d: Cystocele 11: Pelvic muscle training programs are most useful for what type of incontinence? a: Psychogeni c b: Overflo w c: Urg e d: Stress 12: What is the purpose of Kegel exercises? a: Improve bladder capacity and control b: Increase bladder awareness c: Strengthen pelvic floor muscles d: Tighten uterine ligaments 13: A 45-year-old patient is undergoing nonsurgical management of genitourinary problems related to her recent pregnancy. Kegel exercises may be useful in a patient with a: dyspareuni a b: mild stress incontinence c: symptomatic rectocele d: second-degree prolapse of the uterus 14: What is the primary purpose of a ring pessary? a: Decrease bladder capacity b: Obstruct the urethra c: Channel intra-abdominal pressure toward the introitus d: Provide mechanical support 15: Which of the following surgical procedures obliterates the vaginal canal? a: Sacral colpopexy b: Colpocleis is c: Posterior colporrhaphy d: Burch procedure 16: What percent of women will suffer a urinary tract infection at some point in their lives? a: 90% b: 10% c: 60% d: 30% 17: Asymptomatic bacteriuria is found in what percent of postmenopausal women? a: 10% b: <5% c: 30% d: 20% 18: A 30-year-old patient is evaluated for a possible urinary tract infection. The culture of a urine sample is reported to show greater than 100,000 colonies of "mixed flora." This is most likely indicative of a: trigoniti s b: upper urinary tract infection c: a contaminated specimen d: infection of the proximal urethra 19: In a symptomatic patient, which of the following is indicative of lower urinary tract infection? a: >100,000 colonies of mixed flora b: 1,000 colonies of Bacteroides species c: 10,000 colonies of Staphylococcus aureus d: 10,000 colonies of Escherichia coli 20: Cystocele is best defined as a: descent or prolapse of the rectum b: descent or prolapse of the urethra c: descent or prolapse of the bladder d: herniation of the top of the vagina 21: Which of the following is the best description of a rectocele? a: herniation of the top of the vagina b: descent or prolapse of the rectum c: descent or prolapse of the urethra d: descent or prolapse of the bladder 22: Procidentia describes uterine descent beyond the a: vulv a b: ischial spines c: level of the uterine artery d: plane of the pelvic inlet 23: What is the stage of prolapse when the leading part of the prolapse is more than 1 cm beyond the hymen but less than or equal to the total vaginal length? a: I b: III c: II d: IV 24: What is the condition in which the bladder forces the anterior vaginal wall down and out? a: Cystocel e b: Enterocel e c: Rectocel e d: Procidentia 25: Which of the following is the best way to identify a urethral diverticulum? a: Multichannel urodynamic testing b: The Q-tip test c: Cystourethroscop y d: Single-channel urodynamic testing ANS:[ ccbca dabdc dcbdb bcdcd babac ] Chapter 17 Breast Health Linda Christinsen-Rengel 1. While conducting a physical examination on a female adolescent client, you note that glandular tissue has developed under the areola of her breast and that the nipple and breast project as a single mound. At which Tanner phase is this client? A. 1 B. 2 C. 3 D. 4 Answer: B 2. Tamara is a 7-year-old client who is already showing signs of breast development and growth of pubic hair. Which condition does Tamara most likely have? A. Premature telarche B. Amastia C. Precocious puberty D. Polythelia Answer: C 3. Denisa is a 48-year-old client who complains of spontaneous discharge of milk from both breasts. She says she is taking valsartan for her blood pressure and bupropion Hcl for depression. A pregnancy test comes back negative, and her prolactin level is 14 ng/mL. What condition does Denisa most likely have? A. Galactorrhea B. Prolactinoma C. Increased vascularity D. Intraductal papilloma Answer: A 4. You are teaching about breast care to Maria, a client who just gave birth and who plans to breastfeed her child. Which instruction should you give her to help prevent nipple fissures? A. Towel dry her breasts immediately on getting out of the shower B. Apply lanolin to her nipples C. Avoid wearing a bra D. Avoid washing the breasts Answer: B 5. You have just completed a clinical breast exam on Casey, a healthy 23-year-old client with no personal or family history of breast cancer. You should advise Casey to have a clinical breast exam how often? A. Every year B. Every 2 years C. Every 3 years D. Every 5 years Answer: C 6. Margaret is a client who has a documented breast cancer (BRCA) 1 mutation and is at a high risk for breast cancer. Which diagnostic procedure is recommended by the American Cancer Society to screen Margaret for breast cancer? A. Ultrasound B. Magnetic resonance imaging C. Mammogram D. Chest x-ray Answer: B 7. Tiffany is a client who has a large cyst in her breast. Which intervention would be most appropriate for resolving the cyst and obtaining a fluid sample to send to the laboratory for cytology studies? A. Core needle biopsy B. Incisional biopsy C. Excisional biopsy D. Fine needle aspiration biopsy Answer: D 8. Miranda is a 25-year-old client who is concerned about her risk for developing breast cancer. Which of the following should you point out to Miranda as risk factors for breast cancer? (Select all that apply.) A. Being 25 years old B. Having a mutation of the BRCA 1 tumor suppressor gene C. Drinking three beers daily D. Menarche at age 13 years E. Body mass index of 35 Answer: B, C, E 9. Katherine is a 30-year-old client who recently learned that she has the BRCA 1 gene mutation and is terrified that she might develop breast cancer. She lost her mother to breast cancer a few years ago and is prepared to take drastic measures to reduce her risk. She has a body mass index of 24, and drinks a glass of wine daily. Which of the following interventions would most effectively lower Katherine’s risk for developing breast cancer? A. Abstaining from alcohol B. Taking tamoxifen, as prescribed C. Losing 20 lb D. Undergoing surgical prophylaxis Answer: D 10. Marcy has been diagnosed with breast cancer. Her tumor is about 3 cm in diameter. What is the T stage of Marcy’s tumor? A. T1 B. T2 C. T3 D. T4 Answer: B 11. Kelly is a client who was recently diagnosed with breast cancer. She has a tumor 6 cm in diameter and several positive nodes outside the axillary nodes, with no distant metastasis. What stage is Kelly’s cancer? A. 1 B. 2 C. 3 D. 4 Answer: C 12. Eva recently underwent a lumpectomy for treatment of her 7-cm tumor. What additional treatment should Eva most expect to undergo in conjunction with the lumpectomy? A. Radiation therapy B. Chemotherapy C. Hormonal therapy D. Biological therapy Answer: A 13. Harriet has been diagnosed with HER2-positive breast cancer. Which treatment would allow targeting of the cancer cells more specifically and thus would likely produce fewer side effects? A. Radiation therapy B. Chemotherapy C. Hormonal therapy D. Biological therapy Answer: D 14. Pamela is a client who is preparing to undergo chemotherapy to treat her breast cancer. What instructions should you give her to help reduce the risk of acquiring infections? (Select all that apply.) A. Avoid having dental work during treatment B. Stay out of large crowds C. Eat a low-calorie diet D. Wash hands frequently E. Get plenty of rest Answer: A, B, D, E 15. Women who are diagnosed with early breast cancer are thought to have what chance of survival? A. 60% B. 70% C. 80% D. 90% Answer: D Chapter 18 The Menopausal Transition Catherine Juve, Valerie T. Cotter, Ellis Quinn Youngkin 1. Which of the following most accurately characterizes the way Western societies view menopause? A. A hormone deficiency disease B. A celebrated rite of passage C. A nonentity D. A milestone in a spiritual journey Answer: A 2. Postmenopausal women are at increased risk for which of the following? (Select all that apply.) A. Heart disease B. Lupus C. Osteoporosis D. Diabetes E. Turner syndrome Answer: A, C, D 3. What is the average age of menopause? A. 45 years B. 48 years C. 51 years D. 54 years Answer: C 4. Which of the following is the correct definition of perimenopause? A. The period of time between menarche and menopause B. The period of time from the onset of menopause-related symptoms to 12 months following the last menstrual period C. The point in time following 12 months of amenorrhea D. A 5-year period of time following a menopause Answer: B 5. Which of the following changes typically occur in perimenopause? (Select all that apply.) A. Increased production of follicles B. Decreased estrogen levels C. Decreased progesterone levels D. Decreased testosterone levels E. Increased blood glucose levels Answer: B, C 6. What effect does smoking have on the timing of the onset of menopause? A. 3-year earlier onset B. 1.5-year earlier onset C. 1.5-year later onset D. 3-year later onset Answer: B 7. Which ethnicity has been associated with a later onset of menopause? A. Hispanic B. African American C. White D. Japanese Answer: D 8. What percentage of women in perimenopause experience minimal to no menopause-related symptoms? A. 3% B. 13% C. 23% D. 33% Answer: D 9. What is the single most common menopause-related symptom among postmenopausal women? A. Irritability B. Sleep disturbances C. Hot flashes D. Fatigue Answer: C 10. Kayla, a 47-year-old client, has been diagnosed with genitourinary syndrome. On the basis of this diagnosis, the nurse recognizes that Kayla most likely experiences which of the following symptoms, associated with this syndrome? (Select all that apply.) A. Heavier menstrual flow B. Vaginal dryness C. Vulvovaginal atrophy D. Hot flashes E. Dyspareunia Answer: B, C, E 11. A nurse practitioner is preparing to perform a physical examination on a client displaying menopauseassociated symptoms. What should the nurse include in the examination? (Select all that apply.) A. Vital signs B. Height and weight measurements C. Endoscopic examination D. Clinical breast examination E. Pelvic examination Answer: A, B, D, E 12. Daphne is a 49-year-old perimenopausal client who has been experiencing severe hot flashes in recent months. To help Daphne alleviate this symptom, the nurse should recommend that she avoid which of the following? (Select all that apply.) A. Caffeine B. Alcohol C. Dairy products D. Spicy foods E. Meats Answer: A, B, D 13. Nova, a 47-year-old perimenopausal client, has heard that isoflavones can help reduce the severity of hot flashes. Which foods should the nurse mention as containing isoflavones? (Select all that apply.) A. Ginger B. Red clover C. Ginseng D. Soy E. Alfalfa Answer: B, D 14. Lyla, a 45-year-old client who recently entered perimenopause, tells her nurse that she has seen “natural” hormones advertised and asks about them. Which of the following should the nurse tell Lyla about these products? (Select all that apply.) A. Unlike bioidentical hormones, they are derived from all-natural sources B. They cause fewer side effects than do pharmaceutically manufactured hormones C. They are identical to the hormones produced in a woman’s body D. They are not as effective as bioidentical hormones E. They are safer than pharmaceutically manufactured hormones Answer: D 15. The nurse is providing counseling to Renee, a 37-year-old client who has entered perimenopause prematurely. Which of the following should the nurse tell Renee to expect as a result of this condition, compared with women who enter perimenopause at the typical time? (Select all that apply.) A. Less severe menopause-related symptoms B. Early loss of fertility C. Increased risk for osteoporosis D. Decreased risk for cardiovascular disease E. Increased life expectancy Answer: B, C Chapter 19 Promotion of Women’s Health Care During Pregnancy The United States ranks 50th in the world for maternal mortality and 41st among industrialized 1. nations for infant mortality rate. When developing programs to assist in decreasing these rates, which factor would most likely need to be addressed as having the greatest impact? A) Resolving all language and cultural differences B) Assuring early and adequate prenatal care C) Providing more extensive womens shelters D)Encouraging all women to eat a balanced diet 2. When integrating the principles of family-centered care, the nurse would include which of the following? A) Childbirth is viewed as a procedural event B) Families are unable to make informed choices C) Childbirth results in changes in relationships D) Families require little information to make appropriate decisions When preparing a teaching plan for a group of first-time pregnant women, the nurse expects to 3. review how maternity care has changed over the years. Which of the following would the nurse include when discussing events of the 20th century? A) Epidemics of puerperal fever B) Performance of the first cesarean birth C) Development of the x-ray to assess pelvic size D) Creation of free-standing birth centers 4. After teaching a group of students about pregnancy-related mortality, the instructor determines that additional teaching is needed when the students identify which condition as a leading cause? A) Hemorrhage B) Embolism C) Obstructed labor D)Infection The nurse is working with a group of community health members to develop a plan to address 5. the special health needs of women. Which ofthe following conditions would the group address as the major problem? A) Smoking B) Heart disease C) Diabetes D)Cancer 6. When assessing a family for possible barriers to health care, the nurse would consider which factor to be most important? A) Language B) Health care workers attitudes C) Transportation D)Finances After teaching a group of nursing students about the issue of informed consent. Which of the 7. following, if identified by the student, would indicate an understanding of a violation of informed consent? A) Performing a procedure on a 15-year-old without consent B) Serving as a witness to the signature process C) Asking whether the client understands what she is signing D) Getting verbal consent over the phone for emergency procedures The nurse is trying to get consent to care for an 11-year-old boy with diabetic ketoacidosis. His 8. parents are out of town on vacation, and the child is staying with a neighbor. Which action would be the priority? A) Getting telephone consent with two people listening to the verbal consent B) Providing emergency care without parental consent C) Contacting the childs aunt or uncle to obtain their consent D)Advocating for termination of parental rights for this situation 9. After teaching nursing students about the basic concepts of family-centered care, the instructor determines that the teaching was successful when the students state which of the following? A) Childbirth affects the entire family, and relationships will change. B) Families are not capable of making health care decisions for themselves. C) Mothers are the family members affected by childbirth. D)Childbirth is a medical procedure. A nursing instructor is preparing a class discussion on the trends in health care and health care 10.delivery over the past several centuries. When discussing the changes during the past century, which of the following would the instructor be least likely to include? A) Disease prevention B) Health promotion C) Wellness D) Analysis of morbidity and mortality A nurse is assigned to care for an Asian American client. The nurse develops a plan of care with 11.the understanding that based on this clients cultural background, the client most likely views illness as which of the following? A) Caused by supernatural forces. B) A punishment for sins. C) Due to spirits or demons. D) From an imbalance of yin and yang A nurse is developing a plan of care for a woman to ensure continuity of care during pregnancy, 12.labor, and childbirth. Which of the following would be most important for the nurse to incorporate into that plan? A) Adhering to strict, specific routines B) Involving a pediatric physician C) Educating the client about the importance of a support person D) Assigning several nurses as a support team A nursing instructor is preparing a class discussion on case management in maternal and 13.newborn health care. Which of the following would the instructor include as a key component? Select all that apply. A) Advocacy B) Coordination C) Communication D) Resource management E) Event managed care 14.After teaching a group of students about the concept of maternal mortality, the instructor determines that additional teaching is needed when the students state which of the following? A) The rate includes accidental causes for deaths. B) It addresses pregnancy-related causes. C) The duration of the pregnancy is not a concern. D) The time frame is typically for a specified year. A group of students are reviewing the historical aspects about childbirth. The students 15.demonstrate understanding of the information when they identify the use of twilight sleep as a key event during which time frame? A) 1700s B) 1800s C) 1900s D) 2000s 16.A nurse is providing care to a woman who has just delivered a healthy newborn. Which action would least likely demonstrate application of the concept of family-centered care? A) Focusing on the birth as a normal healthy event for the family B) Creating opportunities for the family to make informed decisions C) Encouraging the woman to keep her other children at home D) Fostering a sense of respect for the mother and the family 17.When discussing fetal mortality with a group of students, a nurse addresses maternal factors. Which of the following would the nurse most likely include? Select all that apply. A) Chromosomal abnormalities B) Malnutrition C) Preterm cervical dilation D) Underlying disease condition E) Poor placental attachment A nurse is preparing a presentation for a local community group about health status and 18.childrens health. Which of the following would the nurse include as one of the most significant measures? A) Fetal mortality rate B) Neonatal mortality rate C) Infant mortality rate D) Maternal mortality rate A group of students are reviewing an article describing information related to indicators for 19.womens health and the results of a national study. Which of the following would the students identify as being satisfactory for women? Select all that apply. A) Smoking cessation B) Colorectal cancer screening C) Violence against women D) Health insurance coverage E) Mammograms A nurse is preparing a presentation for a local womens group about heart disease and women. 20.Which of the following would the nurse expect to address when discussing measures to promote health. A) Women have similar symptoms as men for a heart attack. B) Heart disease is no longer viewed as a mans disease. C) Women experiencing a heart attack are at greater risk for dying. D) Heart attacks in women are more easily diagnosed. A nurse is working to develop a health education program for a local community to address 21.breast cancer awareness. Which of the following would the nurse expect to include when describing this problem to the group? Select all that apply. A) White women have higher rates of breast cancer than African American women. B) African American women are more likely to die from breast cancer at any age. C) Survival at any stage is worse among white women. D) Women living in South America have the highest rates of breast cancer. E) Breast cancer is the leading cause of cancer mortality in women. A group of nursing students are reviewing information about factors affecting maternal, 22.newborn, and womens health. The students demonstrate understanding of the information when they identify which of the following deficiencies as being associated with poverty? Select all that apply. A) Literacy B) Employment opportunities C) Mobility D) Political representation E) Skills Answer Key 1. B 2. C 3. D 4. B 5. B 6. D 7. A 8. A 9. A 10.D 11.D 12.C 13.A, B, C, D 14.A 15.C 16.C 17.B, C, D 18.C 19.B, E 20.C 21. A, B 22. A, B, C, D, E Chapter 20 Maternal Conditions Impacting Risk in Pregnancy Debbie Ringdahl, Melissa Frisvold, Joan Corder-Mabe After teaching a woman who has had an evacuation for a hydatidiform mole (molar pregnancy. 1. about her condition, which of the following statements indicates that the nurses teaching was successful? A) I will be sure to avoid getting pregnant for at least 1 year. B) My intake of iron will have to be closely monitored for 6 months. C) My blood pressure will continue to be increased for about 6 more months. D) I wont use my birth control pills for at least a year or two. 2. Which of the following findings on a prenatal visit at 10 weeks might lead the nurse to suspect a hydatidiform mole? A) Complaint of frequent mild nausea B) Blood pressure of 120/84 mm Hg C) History of bright red spotting 6 weeks ago D) Fundal height measurement of 18 cm 3. A client is diagnosed with gestational hypertension and is receiving magnesium sulfate. Which finding would the nurse interpret as indicating a therapeutic level of medication? A) Urinary output of 20 mL per hour B) Respiratory rate of 10 breaths/minute C) Deep tendons reflexes 2+ D)Difficulty in arousing 4. Upon entering the room of a client who has had a spontaneous abortion, the nurse observes the client crying. Which of the following responses by the nurse would be most appropriate? A) Why are you crying? B) Will a pill help your pain? C) Im sorry you lost your baby. D) A baby still wasnt formed in your uterus. 5. Which of the following data on a clients health history would the nurse identify as contributing to the clients risk for an ectopic pregnancy? A) Use of oral contraceptives for 5 years B) Ovarian cyst 2 years ago C) Recurrent pelvic infections D) Heavy, irregular menses 6. In a woman who is suspected of having a ruptured ectopic pregnancy, the nurse would expect to assess for which of the following as a priority? A) Hemorrhage B) Jaundice C) Edema D)Infection 7. Which of the following findings would the nurse interpret as suggesting a diagnosis of gestational trophoblastic disease? A) Elevated hCG levels, enlarged abdomen, quickening B) Vaginal bleeding, absence of FHR, decreased hPL levels C) Visible fetal skeleton on ultrasound, absence of quickening, enlarged abdomen D)Gestational hypertension, hyperemesis gravidarum, absence of FHR 8. It is determined that a clients blood Rh is negative and her partners is positive. To help prevent Rh isoimmunization, the nurse anticipates that the client will receive RhoGAM at which time? A) At 34 weeks gestation and immediately before discharge B) 24 hours before delivery and 24 hours after delivery C) In the first trimester and within 2 hours of delivery D) At 28 weeks gestation and again within 72 hours after delivery The nurse is developing a plan of care for a woman who is pregnant with twins. The nurse 9. includes interventions focusing on which of the following because of the womans increased risk? A) Oligohydramnios B) Preeclampsia C) Post-term labor D)Chorioamnionitis A woman hospitalized with severe preeclampsia is being treated with hydralazine to control 10.blood pressure. Which of the following would the lead the nurse to suspect that the client is having an adverse effect associated with this drug? A) Gastrointestinal bleeding B) Blurred vision C) Tachycardia D) Sweating 11.After reviewing a clients history, which factor would the nurse identify as placing her at risk for gestational hypertension? A) Mother had gestational hypertension during pregnancy. B) Client has a twin sister. C) Sister-in-law had gestational hypertension. D) This is the clients second pregnancy. A client with hyperemesis gravidarum is admitted to the facility after being cared for at home 12.without success. Which of the following would the nurse expect to include in the clients plan of care? A) Clear liquid diet B) Total parenteral nutrition C) Nothing by mouth D) Administration of labetalol The nurse is reviewing the laboratory test results of a pregnant client. Which one of the following findings would alert the nurse to the development of HELLP syndrome? A) Hyperglycemia B) Elevated platelet count C) Leukocytosis D) Elevated liver enzymes Which of the following would the nurse have readily available for a client who is receiving magnesium sulfate to treat severe preeclampsia? A) Calcium gluconate B) Potassium chloride C) Ferrous sulfate D) Calcium carbonate Which assessment finding would lead the nurse to suspect infection as the cause of aclients PROM? A) Yellow-green fluid B) Blue color on Nitrazine testing C) Ferning D) Foul odor While assessing a pregnant woman, the nurse suspects that the client may be at risk for hydramnios based on which of the following? (Select all that apply.) A) History of diabetes B) Complaints of shortness of breath C) Identifiable fetal parts on abdominal palpation D) Difficulty obtaining fetal heart rate E) Fundal height below that for expected gestataional age After teaching a group of nursing students about the possible causes of spontaneous abortion, 17.the instructor determines that the teaching was successful when the students identify which of the following as the most common cause of first trimester abortions? A) Maternal disease B) Cervical insufficiency C) Fetal genetic abnormalities D) Uterine fibroids A pregnant woman is admitted with premature rupture of the membranes. The nurse is assessing 18.the woman closely for possible infection. Which of the following would lead the nurse to suspect that the woman is developing an infection? (Select all that apply.) A) Fetal bradycardia B) Abdominal tenderness C) Elevated maternal pulse rate D) Decreased C-reactive protein levels E) Cloudy malodorous fluid A nurse is teaching a pregnant woman with preterm premature rupture of membranes who is 19.about to be discharged home about caring for herself. Which statement by the woman indicates a need for additional teaching? A) I need to keep a close eye on how active my baby is each day. B) I need to call my doctor if my temperature increases. C) Its okay for my husband and me to have sexual intercourse. D) I can shower but I shouldnt take a tub bath. 20.A nurse is assessing a pregnant woman with gestational hypertension. Which of the following would lead the nurse to suspect that the client has developed severe preeclampsia? A) Urine protein 300 mg/24 hours B) Blood pressure 150/96 mm Hg C) Mild facial edema D) Hyperreflexia 21.A nurse suspects that a pregnant client may be experiencing abruption placenta based on assessment of which of the following? (Select all that apply.) A) Dark red vaginal bleeding B) Insidious onset C) Absence of pain D) Rigid uterus E) Absent fetal heart tones The health care provider orders PGE2 for a woman to help evacuate the uterus following a spontaneous abortion. Which of the following would be most important for the nurse to do? A) Use clean technique to administer the drug. B) Keep the gel cool until ready to use. C) Maintain the client for hour after administration. D) Administer intramuscularly into the deltoid area. A nursing student is reviewing an article about preterm premature rupture of membranes. 23. Which of the following would the student expect to find as factor placing a woman at high risk for this condition? (Select all that apply.) A) High body mass index B) Urinary tract infection C) Low socioeconomic status D) Single gestations E) Smoking A woman with placenta previa is being treated with expectant management. The woman and 24.fetus are stable. The nurse is assessing the woman for possible discharge home. Which statement by the woman would suggest to the nurse that home care might be inappropriate? A) My mother lives next door and can drive me here if necessary. B) I have a toddler and preschooler at home who need my attention. C) I know to call my health care provider right away if I start to bleed again. D) I realize the importance of following the instructions for my care. 25.A woman with hyperemesis gravidarum asks the nurse about suggestions to minimize nausea and vomiting. Which suggestion would be most appropriate for the nurse to make? A) Make sure that anything around your waist is quite snug. B) Try to eat three large meals a day with less snacking. C) Drink fluids in between meals rather than with meals. D) Lie down for about an hour after you eat 26.A woman with gestational hypertension experiences a seizure. Which of the following would be the priority? A) Fluid replacement B) Oxygenation C) Control of hypertension D) Delivery of the fetus A woman is receiving magnesium sulfate as part of her treatment for severe preeclampsia. The 27.nurse is monitoring the womans serum magnesium levels. Which level would the nurse identify as therapeutic? A) 3.3 mEq/L B) 6.1 mEq/L C) 8.4 mEq/L D) 10.8 mEq/L Answer Key 1. A 2. D 3. C 4. C 5. C 6. A 7. D 8. D 9. B 10.C 11.A 12.C 13.D 14.A 15.D 16.A, B, D 17.C 18.B, C, E 19.C 20.D 21.A, D, E 22.C 23.B, C, E 24.B 25.C 26.B 27.B Chapter 21 Assessing Fetal Well-Being Michele Davidson, Marion Herndon Fuqua While talking with a pregnant woman who has undergone genetic testing, the woman informs 1. the nurse that her baby will be born with Down syndrome. The nurse understands that Down syndrome is an example of: A) Multifactorial inheritance B) X-linked recessive inheritance C) Trisomy numeric abnormality D)Chromosomal deletion 2. A nurse is describing advances in genetics to a group of students. Which of the following would the nurse least likely include? A) Genetic diagnosis is now available as early as the second trimester. B) Genetic testing can identify presymptomatic conditions in children. C) Gene therapy can be used to repair missing genes with normal ones. D) Genetic agents may be used in the future to replace drugs. After teaching a group of students about fetal development, the instructor determines that the 3. teaching was successful when the students identify which of the following as providing the barrier to other sperm after fertilization? A) Zona pellucida B) Zygote C) Cleavage D)Morula 4. A nurse is teaching a class on X-linked recessive disorders. Which of the following statements would the nurse most likely include? A) Males are typically carriers of the disorders. B) No male-to-male transmission occurs. C) Daughters are more commonly affected with the disorder. D) Both sons and daughters have a 50% risk of the disorder. A pregnant woman undergoes maternal serum alpha-fetoprotein (MSAFP) testing at 16 to 18 5. weeks gestation. Which of the following would the nurse suspect if the womans level is decreased? A) Down syndrome B) Sickle-cell anemia C) Cardiac defects D) Open neural tube defect The nurse is developing a presentation for a community group of young adults discussing fetal 6. development and pregnancy. The nurse would identify that the sex of offspring is determined at the time of: A) Meiosis B) Fertilization C) Formation of morula D)Oogenesis 7. When describing amniotic fluid to a pregnant woman, the nurse would include which of the following? A) This fluid acts as transport mechanism for oxygen and nutrients. B) The fluid is mostly protein to provide nourishment to your baby. C) This fluid acts as a cushion to help to protect your baby from injury. D) The amount of fluid remains fairly constant throughout the pregnancy. 8. Assessment of a pregnant woman reveals oligohydramnios. The nurse would be alert for the development of which of the following? A) Maternal diabetes B) Placental insufficiency C) Neural tube defects D) Fetal gastrointestinal malformations A couple comes to the clinic for preconception counseling and care. As part of the visit, the 9. nurse teaches the couple about fertilization and initial development, stating that the zygote formed by the union of the ovum and sperm consists of how many chromosomes? A) 22 B) 23 C) 44 D)46 10.A woman just delivered a healthy term newborn. Upon assessing the umbilical cord, the nurse would identify which of the following as normal? (Select all that apply.) A) One vein B) Two veins C) One artery D) Two arteries E) One ligament F) Two ligaments After teaching a pregnant woman about the hormones produced by the placenta, the nurse 11.determines that the teaching was successful when the woman identifies which hormone produced as being the basis for pregnancy tests? A) Human placental lactogen (hPL) B) Estrogen (estriol) C) Progesterone (progestin) D) Human chorionic gonadotropin (hCG) After the nurse describes fetal circulation to a pregnant woman, the woman asks why her fetus 12.has a different circulation pattern than hers. In planning a response, the nurse integrates understanding of which of the following? A) Fetal blood is thicker than that of adults and needs different pathways. B) Fetal circulation carries highly oxygenated blood to vital areas first. C) Fetal blood has a higher oxygen saturation and circulates more slowly. D) Fetal heart rates are rapid and circulation time is double that of adults. 13.When describing genetic disorders to a group of childbearing couples, the nurse would identify which as an example of an autosomal dominant inheritance disorder? A) Huntingtons disease B) Sickle cell disease C) Phenylketonuria D) Cystic fibrosis Prenatal testing is used to assess for genetic risks and to identify genetic disorders. In explaining 14.to a couple about an elevated alpha-fetoprotein screening test result, the nurse would discuss the need for: A) Special care needed for a Down syndrome infant B) A more specific determination of the acidbase status C) Further, more definitive evaluations to conclude anything D) Immediate termination of the pregnancy based on results A nursing instructor is preparing a teaching plan for a group of nursing students about the 15.potential for misuse of genetic discoveries and advances. Which the following would the instructor most likely include? A) Gene replacement therapy for defective genes B) Individual risk profiling and confidentiality C) Greater emphasis on the causes of diseases D) Slower diagnosis of specific diseases After teaching a class on the stages of fetal development, the instructor determines that the 16.teaching was successful when the students identify which of the following as a stage? (Select all that apply.) A) Placental B) Preembryonic C) Umbilical D) Embryonic E) Fetal A nurse is discussing fetal development with a pregnant woman. The woman is 12 weeks 17.pregnant and asks, Whats happening with my baby? Which of the following would the nurse integrate into the response? (Select all that apply.) A) Continued sexual differentiation B) Eyebrows forming C) Startle reflex present D) Digestive system becoming active E) Lanugo present on the head After teaching a group of students about fetal development, the instructor determines that the 18.teaching was successful when the students identify which of the following as essential for fetal lung development? A) Umbilical cord B) Amniotic fluid C) Placenta D) Trophoblasts During a prenatal class for a group of new mothers, the nurse is describing the hormones 19.produced by the placenta. Which of the following would the nurse include? (Select all that apply.) A) Prolactin B) Estriol C) Relaxin D) Progestin E) Human chorionic somatomammotropin 20.When describing the structures involved in fetal circulation, the nursing instructor describes which structure as the opening between the right and left atrium? A) Ductus venosus B) Foramen ovale C) Ductus arteriosus D) Umbilical artery A group of students are reviewing information about genetic inheritance. The students 21.demonstrate understanding of the information when they identify which of the following as an example of an autosomal recessive disorder? (Select all that apply.) A) Cystic fibrosis B) Phenylketonuria C) Tay-Sachs disease D) Polycystic kidney disease E) Achondroplasia 22.A nurse is assessing a child with Klinefelters syndrome. Which of the following would the nurse expect to assess? (Select all that apply.) A) Gross mental retardation B) Long arms C) Profuse body hair D) Gynecomastia E) Enlarged testicles 23.A woman is scheduled to undergo fetal nuchal translucency testing. Which of the following would the nurse include when describing this test? A) A needle will be inserted directly into the fetuss umbilical vessel. B) Youll have an intravaginal ultrasound to measure fluid in the fetus. C) The doctor will take a sample of fluid from your bag of waters. D) A small piece of tissue from the fetal part of the placenta is taken. Answer Key 1. C 2. A 3. A 4. B 5. A 6. B 7. C 8. B 9. D 10.A 11.D 12.B 13.A 14.C 15.B 16. B, C, E 17. A, D 18.B 19.B, C, D, E 20.B 21.A 22.B, D 23.B Chapter 22 Postpartum and Lactation Diane Schadewald, Cheri Friedrich, Kathleen M. Akridge Your clinic nurse warns you that your next patient may take some time. Upon entering the room you find a tearful 16-year-old G1P1 who presents for her 6-week postpartum visit. 1: The puerperium, the period following birth during which the reproductive tract returns to its normal, nonpregnant state, lasts approximately a: 14-16 weeks b: 10-12 weeks c: 6-8 weeks d: 2-4 weeks e: 18-20 weeks During your interview the patient expresses concern that she and her partner attempted intercourse several weeks ago. It was painful and she is afraid there may be something wrong. 2: The best treatment for dyspareunia in the post partum period is a: analgesic b: antidepressan t c: oral estrogen d: lubrican t e: topical estrogen Is it possible that she could get pregnant? She is not breast-feeding. 3: A. The mean time to ovulation in the nonlactating postpartum woman is approximately a: 75 days b: 45 days c: 60 days d: 15 days e: 30 days 4: B. Approximately of non-nursing women are fertile at 6-week postpartum. a: 10 % b: 15 % c: 20% d: 25% e: 5% The patient's crying escalates. This was an unplanned pregnancy and she is terrified of being pregnant again. She is grouchy and moody all day and things just get worse at night with the baby getting colicky after the formula, her boyfriend just watching TV, and her mother giving unwanted advice. She has not slept in days. 5: A. The incidence of postpartum blues is approximately . a: 50%-60% b: 70%-80% c: 90%-99% d: 30%-40% e: 10%-20% 6: B. Approximately what percent of new mothers experience postpartum depression? a: 20%-25% b: 1%-5% c: 10%-15% d: 30%-45% e: 50%-65% 7: C. The incidence of postpartum psychosis is approximately a: 0.05%-0.1% b: 0.7%-0.8% c: 0.1%-0.2% d: 0.5%-0.6% e: 0.3%-0.4% During your review of systems she admits to leaking urine during coughing or sneezing. 8: A. The best treatment fo postpartum stress incontinence is a: topical estrogen medication b: empirical antibiotic treatment for cystitis c: anticholinergic medication d: tricyclic antidepressant medication e: reassurance and re-evaluation in 2 months 9: B. The return of normal tone to the pelvic floor muscles postpartum may be enhanced by a: exogenous estrogen supplementation b: Kegel exercises c: avoiding coitus fo 3 to 6 months d: endogenous estrogen supplementation e: endogenous progesterone supplementation On continuation of your review of systems she also admits to bleeding with bowel movements and she thinks it is due to the hemorrhoids she developed during her 2 hours of pushing. 10: Surgical management of postpartum hemorrhoids may be considered how soon postpartum? a: 3 months b: 18 months c: 6 months d: 12 months e: Immediately Your physical exam is within normal limits, including normal thyroid, heart, lungs, and breast exam. On pelvic exam you find that vaginal mucosa is intact but moderately atrophic and her uterus is anteverted, small, and firm with minimal lochia. She is noted to have some nonthrombosed hemorrhoids. 11: A. Uterine involution is a result of a decrease in the a: size of the intercellular spaces in the uterine myometrium b: increased intravascular volume, especially venous space volume c: number of cells in the uterine myometrium d: number of intercellular spaces in the uterine myometrium e: size of cells in the uterine myometrium 12: B. How many weeks does it take for the uterus to return to its pre-pregnancy position in the true pelvis? a: 2 b: 4 c: 8 d: 10 e: 6 13: C. How many weeks does it take for the uterus to return to its pre-pregnancy size? a : 2 b : 6 c : 8 d: 1 0 e : 4 You receive a call from the Labor and Delivery nurse. The patient you delivered by forceps 3 hours ago is bleeding more than usual. Her pulse is now 110 bpm and she is rapidly soaking pads. 14: A. Immediate postpartum uterine hemostasis is maintained by a: primary clotting of blood in the uterine artery b: tamponade effect of clots c: scar formation within the uterine cavity d: contraction of the uterine smooth muscle e: decrease in myometrial cell volume 15: B. The elevated pulse rate characteristic of pregnancy a: decreases approxiamately 1 hour after delivery b: decreases approximately 6 hours after delivery c: persists for approximately 3 weeks postpartum d: decreases at the end of the third stage of labor e: persists for about 3 months postpartum On exam you notice that her bladder is full and she is unable to void spontaneously, so a Foley catheter is inserted and 600 mL of urine drained from the bladder. 16: Transitory urinary retention in the postpartum period following vaginal delivery is primarily related to a: peripartum cystitis b: periurethral edema c: parasympathomimetic discharge d: sympathomimetic discharge e: progesterone-associated loss of bladder contractility On exam, there are no cervical or vaginal lacerations and the bleeding responds to fundal massage and an increase in the Pitocin infusion rate. You recommend the patient to proceed with breast-feeding and assure the patient and nurse that you will follow-up in a few hours. 17: A. Postpartum uterine contractile pain is greater in breast-feeding women bcause suckling releases a: oxytocin b: estroge n c: progesteron e d: prostaglandin s e: inhibin 18: B. When is the endometrium re-established after delivery in most patients? a: First week b: Fourth week c: Third week d: Second week e: Fifth week You return later and find the bleeding has stabilized. The urine output is 100 to 200 mL per hour. 19: A. In a normal patient immediately after delivery, what is the weight of fluid lost through diuresis and loss of extravascular fluid? a: 3 k g b: 7 k g c: 9 k g d: 1 k g e: 5 kg 20: B. In the intrapartum and postpartum periods, the glomerular filtration rate decreases tonormal a: 1-2 days after delivery b: at the time of delivery c: 3-5 days after delivery d: several weeks after delivery e: at the onset of labor ANS: [c,e,b,b,b,c,c,e,b,c,c,a,b,d,a,b,d,a,b,a,c,e,d] IV Primary Care Conditions Affecting Women’s Health Chapter 23 Common Medical Problems: Cardiovascular through Hematological Disorders Mary Benbenek, Mary Dierich, Elaine Ferrary, Rita A. Seeger Jablonski 1. On average, how much later in women than in men does cardiovascular disease typically occur? A. 2 to 4 years B. 6 to 8 years C. 10 to 12 years D. 14 to 16 years Answer: C 2. The lifetime risk at the age of 50 years of women developing cardiovascular disease is which of the following? A. 9.2% B. 19.2% C. 29.2% D. 39.2% Answer: D 3. Which of the following are modifiable risk factors associated withcardiovascular disease? (Select all that apply.) A. Male gender B. Age of 75 years C. Cigarette smoking D. Body mass index (BMI) of 31 E. Moderate exercise six times per year Answer: C, D, E 4. Which ofthe following are psychosocial risk factors associated with cardiovascular disease? (Select all that apply.) A. Obsessive-compulsive disorder B. Depression C. Anxiety D. Premenstrual dysphoric disorder E. Emotional stress Answer: B, C, E 5. The nurse must screen a client for the presence of cardiovascular disease (CVD). For which of the following conditions is EKG-stress testing on a treadmill indicated? (Select all that apply.) A. High risk for CVD B. Symptoms of typical angina C. Ability to achieve five metabolic equivalents (METs) of a task D. Abnormal baseline EKG E. Family history of CVD Answer: B, C, E 6. Claire, a 45-year-old client, is suspected of having atherosclerosis, and the nurse would like to have her undergo diagnostic testing that will detect and quantify the amount of calcium in her arteries. Which diagnostic test should the nurse select? A. Exercise electrocardiogram stress testing B. Computed tomography of the coronary vasculature C. Cardiovascular magnetic resonance imaging D. Coronary angiography and cardiac catheterization Answer: B 7. The nurse takes a blood pressure reading of a 52-year-old client, Hazel, and finds that it is 144/96 mmHg. Hazel has a body mass index (BMI) of 18, smokes, and is sedentary. Which of the following lifestyle recommendations should the nurse make to this client? (Select all that apply.) A. Lose 10 lb B. Limit sodium intake to 2,400 mg/d C. Quit smoking D. Engage in moderate-to-vigorous aerobic physical activity at least 3 to 4 sessions/wk, for 40 minutes per session E. Increase intake of red meat Answer: B, C, D 8. Maisie, a 78-year-old client, reports feeling tightness and pain in her chest that occurs when she climbs stairs or walks any distance when shopping. Her symptoms go away after she rests for a few minutes. What condition should the nurse suspect most in Maisie, based on these symptoms? A. Transient ischemic attack B. Myocardial infarction C. Atrial fibrillation D. Angina pectoris Answer: D 9. Roberta is a 69-year-old client who reports having had an “episode” recently. She describes temporary symptoms of numbness in her right side, confusion, and difficulty in speaking. She says that she was planning on coming to the hospital about it, but the symptoms cleared up in about 20 minutes and she feel fine now. Which condition should the nurse suspect most in Roberta? A. Stroke B. Transient ischemic attack C. Angina pectoris D. Atrial fibrillation Answer: B 10. Women are more likely than men to experience which of the following symptoms in an acute myocardial infarction? (Select all that apply.) A. Chest pain B. Nausea C. Shortness of breath D. Cold sweats E. Dizziness Answer: B, C, D, E 11. A nurse is caring for a client who just presented with signs of stent thrombosis (ST)-elevation myocardial infarction. An emergency angioplasty with stenting procedure has been planned for the client. The nurse knows that this procedure must be initiated on this client within how long since her first contact with the nurse? A. 30 minutes B. 60 minutes C. 90 minutes D. 120 minutes Answer: C 12. The nurse is evaluating a client diagnosed with heart failure, who reports severe, persistent symptoms while at rest. Which functional class applies to this client? A. Class I B. Class II C. Class III D. Class IV Answer: D 13. Candace, an 86-year-old client, reports a sensation of palpitations, heart racing, and heart flutters in her chest, as well as fatigue and shortness of breath. She was diagnosed with Class II heart failure 6 months ago. Which condition should the nurse suspect most in Candace? A. Atrial flutter B. Angina pectoris C. Transient ischemic attack D. Mitral valve prolapse Answer: A 14. When auscultating a client’s heart, the nurse hears a high-pitched midsystolic click atthe apex of the heart. What condition should the nurse associate with this finding? A. Mitral regurgitation B. Mitral valve prolapse C. Mitral stenosis D. Aortic regurgitation Answer: B 15. Which of the following is the most common type of cardiomyopathy and the third leading cause of heart failure in the United States after coronary artery disease and hypertension? A. Dilated B. Hypertrophic C. Restrictive D. Arrhythmogenic right ventricular Answer: A Chapter 24 Common Medical Problems: Musculoskeletal Injuries through Urinary Tract Disorders Gwen Short, Rita A. Seeger Jablonski, Elaine Ferrary 1. Which of the following are factors that increase a person’s risk for pelvic organ prolapse? (Select all that apply.) A. Female B. Age of 78 years C. Nulliparity D. Body mass index (BMI) of 23 E. Smoker Answer: A, B, E 2. Which of the following is a significant downward rotation of more than 35° of thedistal anterior vaginal wall caused by a defect in the suburethral connective tissue and pubourethral (pubovisceral) ligaments? A. Anterior vaginal wall prolapse B. Hypermobile urethra C. Apical vaginal prolapse D. Posterior vaginal wall prolapse Answer: B 3. Misbah is a 69-year-old client who has just been diagnosed with pelvic organ prolapse. She reports vaginal pain when working in her garden and describes symptoms indicating urinary retention. On performing a pelvic exam, the nurse observes that the client’s vagina appears dry and shows signs of erosion and ulceration. On the basis of these symptoms, which stage of pelvic organ prolapse is Misbah most likely in? A. Stage 1 B. Stage 2 C. Stage 3 D. Stage 4 Answer: D 4. Tierra has been diagnosed with Stage 2 pelvic organ prolapse. Which of the following symptoms should the nurse expect to find in this client? (Select all that apply.) A. Lower back pain B. Pelvic heaviness or fullness C. Obstructive urination D. Pain during intercourse E. Urinary retention Answer: A, B, D 5. Alex has been diagnosed with Stage 3 pelvic organ prolapse. She has been experiencing urinary incontinence and is in need of vaginal support. Which of the following interventions would fully meet the needs of this client, most? A. Pelvic floor muscle therapy B. Donut pessary C. Neurologic stimulation therapy D. Estrogen therapy Answer: B 6. Which treatment for pelvic organ prolapse involves a prescriptive medical device that combines Kegel exercises and noninvasive electrical stimulation via biphasic wave forms to stimulate pelvic nerves to enhance contractile response? A. Intone B. Percutaneous tibia nerve simulation C. Pessary therapy D. Pelvic floor muscle therapy Answer: A 7. Which of the following symptoms are associated with painful bladdersyndrome/interstitial cystitis? (Select all that apply.) A. Urinary retention B. Urethral pain C. Sensation of organs descending into the vagina D. Urinary urgency E. Nocturia Answer: B, D, E 8. Which of the following is typically the cause of urinary frequency in clients with painful bladder syndrome/interstitial cystitis? A. Fear of urinary urge incontinence B. Anxiety over developing a urinary tract infection C. Desire to relieve pain by voiding D. Sensation of urinary retention Answer: C 9. Tiffany, a client suspected of having painful bladder syndrome/interstitial cystitis, has been found to have hematuria. Tiffany has a history of surgical mesh augmentation for the correction of an anterior pelvic defect. Which diagnostic test would be best to rule out an alternative condition in this client? A. Potassium sensitivity test B. Symptom scale C. Blood glucose level D. Cystoscopy with hydrodistention Answer: D 10. Isabelle is suspected of having painful bladder syndrome/interstitial cystitis as a result of a neurological hypersensitivity. Given the suspected etiology in this case, which treatment would be most appropriate for Isabelle? A. Pentosan polysulfate sodium B. Amitriptyline C. Montelukast D. Cyclosporine A Answer: B 11. Valerie is suspected of having painful bladder syndrome/interstitial cystitis as a result of a body-wall abnormality. Given the suspected etiology in this case, which treatment would be most appropriate for Valerie? A. Physical therapy B. Pessary therapy C. Electrical stimulation therapy D. Behavioral therapy Answer: A 12. Kelly is suspected of having painful bladder syndrome/interstitial cystitis as a result of a mast-cell response. Given the suspected etiology in this case, which treatment would be most appropriate for Valerie? A. Pentosan polysulfate sodium B. Amitriptyline C. Montelukast D. Cyclosporine A Answer: C 13. Which of the following are common symptoms of cystitis? (Select all that apply.) A. Low back pain B. Frequent urge to urinate C. An inability to urinate due to obstruction D. A sensation of burning during urination E. Pain with sitting or walking Answer: B, D 14. Which of the following ones typically causes cystitis? A. A defect in the connective tissue supporting the bladder B. Bacteria entering the bladder via the urethra C. An urothelial abnormality D. A virus entering the uterus via the cervix Answer: B 15. What percentage of women older than 55 years experience a recurrence of a urinary tract infection within a year? A. 33% B. 43% C. 53% D. 63% Answer: C Chapter 25 Psychosocial Health Concerns Ann Bateman,Eugenia Zelanko 1. Which of the following is the best definition for mental health, according to an emerging consensus from research findings? A. Absence of psychiatric symptoms B. Well-being C. Happiness D. Self-realization Answer: B 2. The eudaemonic approach to well-being concerns which of the following? A. Happiness and life satisfaction B. Financial and material security C. Physical fitness and health D. Self-realization and meaning Answer: D 3. Phyllis is 55-year-old client who works as a real estate agent while caring for her three teenaged children and her elderly father. Based on Erikson’s theory of psychosocial development, which of the following is the developmental crisis Phyllis faces? A. Intimacy versus isolation B. Integrity versus despair C. Generativity versus stagnation D. Identity versus confusion Answer: C 4. Anwara is a 78-year-old client whose husband died 2 years ago. Her daughter and son-in-law, who live in another state, are urging her to relocate to their area, but Anwara is reluctant to leave the community she has lived in for over 30 years. Based on Erikson’s theory of psychosocial development, which of the following is the developmental crisis Anwara faces? A. Intimacy versus isolation B. Integrity versus despair C. Generativity versus stagnation D. Identity versus confusion Answer: B 5. Keiko is a 32-year-old client who recently graduated from medical school and began a residency in neurosurgery. She loves the career she is embarking on but is single and often feels lonely. Based on Erikson’s theory of psychosocial development, which of the following is the developmental crisis Keiko faces? A. Intimacy versus isolation B. Integrity versus despair C. Generativity versus stagnation D. Identity versus confusion Answer: A 6. Which of the following living arrangements is most characteristic of individuals in the phase of psychosocial development known as emerging adulthood? A. Living independently in a single-family home B. Changing places of residence frequently C. Living with a roommate in an apartment D. Cohabiting with a life partner Answer: B 7. Anna is a 23-year-old client who operates her own pastry business. Based on her phase of psychosocial development, Anna is most likely concerned with establishing commitments in which of the following? (Select all that apply.) A. Political party identification B. Caregiving to a spouse or parent C. Religious beliefs D. Love relationships E. Leaving a legacy to the next generation Answer: A, C, D 8. Olivia is a junior in college who is struggling to decide on a major. She had been an education major because her grandmother, mother, and aunt were all teachers, and she looks up to them. But after taking some introductory-level classes in education, she decided it was just not for her. Now she feels like she is back to square one, and is considering everything from engineering to photography. What is Olivia’s current status related to the process of identity formation? A. Identity diffusion B. Foreclosure C. Moratorium D. Identity achievement Answer: C 9. Lulu is a 30-year-old client who recently gave birth to a son. Lulu is passionate about the benefits of breastfeeding and has spent many hours researching lactation, approaches to breastfeeding, and breast pumps. Which developmental change is Lulu demonstrating? A. Cognitive and emotional development B. Emotional changes and maturation C. Shift in quality of being D. Personal agency Answer: D 10. Maia is a 47-year-old client who in recent years has become a committed conservationist and is determined to protect the existence of open lands and wildlife in her region so that her children and later generations can enjoy the beautiful landscapes she grew with. Maia is demonstrating which quality? A. Generativity B. Integrity C. Autonomy D. Identity Answer: A 11. Denae is a 50-year-old client who identifies herself as a member of a religion primarily because she grew up in a family and a community that was committed to this belief system. She has never really explored other belief systems and finds it difficult to explain why she has adopted her religion. Which process of forming a spiritual identity has Denae most likely chosen? A. Diffused B. Foreclosed C. Achieved D. Moratorium Answer: B 12. Nancy is a 47-year-old client who has worked as a stay-at-home mom for the past 20 years, caring for her three children. As her youngest approaches her senior year in high school, Nancy fears empty nest syndrome. What actions can she take to ease the transition to the postparental period? (Select all that apply.) A. Volunteer with her church’s ministry to the homeless B. Take classes at a local community college in preparation for starting her own business C. Encourage her daughter to stay home and attend college locally D. Take voice lessons with the goal of auditioning for a local choral group E. Adopt a child from another country Answer: A, B, D 13. Gertrude is an 85-year-old client who lives in a long-term care facility. Whenever her grandson visits her, she delights in telling stories about her performances as an actress on the stage decades ago and, occasionally, about her regrets related to a failed marriage. Gertrude is engaging in which developmental process? A. Identity exploration B. Social avoidance C. Life review D. Normative transition Answer: C 14. Sophia is a 78-year-old client who is always upbeat and full of stories about all the accomplishments of her children and grandchildren. It always seems as if everything goes well with her and her family and that she never experiences any disappointments or regrets, although you are aware that one of her children recently divorced and a grandson has been incarcerated. Which integrity status is most appropriate for Sophia? A. Integrated B. Pseudointegrated C. Nonexploratory D. Despairing Answer: B 15. Maggie is a 67-year-old client who will be retiring in 6 months from the law practice she founded 30 years ago. She is currently training one of her colleagues to take over her clients and run the practice after she retires. This strategy for smoothing the transition to retirement is known as which of the following? A. Instrumental support B. Identity consolidation C. Life review D. Succession planning Answer: D Chapter 26 Substance Use Disorders and Women Multiple Choice Identify the choice that best completes the statement or answers the question. 1. Poor impulse control, rapid speech, and hypertension are most characteristic of abusing which substance? A. Alcohol B. Heroin C. Cocaine D. LSD 2. A new patient with a history of alcoholism is in the ER with agitation, vomiting, and tremors. She tells you he had his last drink 24 hours ago. Which medication would most likely be ordered? A. Chlordiazepoxide B. Disulfiram C. Chlorpromazine D. Naloxone hydrochloride 3. Your patient has a long history of alcohol abuse. You know that denial is a frequently used defense mechanism. Which statement is indicative of denial? A. My father was a drinker so I guess that led me to this. B. I can stop anytime I want, I just dont feel like it now. C. Drinking calms my nerves. D. I drink when my kids upset me. 4. Which of the following statements from an alcoholic patient reflects a good understanding of her condition? A. I will stick to wine or beer from now on. B. Ill be OK if my wife will just stop nagging me. C. I plan to take my sobriety 1 day at a time. D. I wont need AA after I am sober for 1 year. 5. Your 42-year-old patient in the alcohol treatment unit tells you she often cant remember events while she was drinking. What is this most likely an example of? A. Denial B. Blackouts C. Psychosis D. Depression 6. A teenager admits to you that she has been smoking marijuana. The nurse knows that marijuana is a(n): A. Cannabinol. B. Amphetamine. C. Hallucinogen. D. Narcotic. 7. Which drug cannot be given if the patient reports alcohol intake in the last 24 hours? A. Chlorpromazine B. Chlordiazepoxide C. Disulfiram D. Risperidone 8. Your patient in the ER waiting room is inebriated. She becomes increasingly loud and abusive while waiting to be seen. What would be the best intervention for the nurse? A. Tell him he has to wait his turn, as others were here first. B. Inform him he will be asked to leave if this behavior continues. C. Offer to take him to an exam room to wait for the doctor. D. Do nothing, as he is still in the waiting room. 9. Your patient tells you her husband has a serious drinking problem. Which statement tells you she may be in a codependent relationship? A. Ive reached my limit with his drinking. B. I called his job and told them he was sick when he couldnt go to work. C. The kids are ashamed of their father. I feel bad about that. D. He is drinking less this week. 10. What should be your response to the wife who says, I should get out of this bad situation with his drinking? A. That happened to me. Its best to get out while you can. B. Tell me more about the bad situation. C. Why dont you talk to your husband about his drinking? D. Youll do whats right. 11. Your new patient is at risk for alcohol withdrawal. You know that alcohol withdrawal tends to develop within what period after the last drink? A. 1 week B. 24 to 48 hours C. 1 hour D. More than 1 week 12. The wife of your alcoholic patient has been attending Al-Anon meetings for the past 2 weeks. Which statement tells you the wife is benefitting from the meetings? A. I can tolerate his destructive behavior now that I see how bad other women have it. B. I realize that I provoke his drinking when I go out with my friends. C. I no longer feel that I have to tolerate his berating me. D. It is great to get out of the house and away from the tension. 13. Which of the following are signs of withdrawal from heroin? A. Insomnia, muscle cramps, vomiting B. Excessive sleeping, low blood pressure, depression C. Seizures, brain damage, excessive sleeping D. Lethargy, panic disorder, increased appetite 14. Ms. Thomas, who is 50 years old, is being treated for pneumonia and dehydration. She has a history of alcoholism and admits to starting to drink heavily again. She tells you, Im a horrible person. My family deserves someone better than me to care for them. Your most therapeutic response to him is: It sounds as though you are feeling guilty about drinking, Ms. Thomas. Tell me more about A. what you mean by that. B. Why do you say that, Ms. Thomas? C. Im sure that your family is satisfied, Ms. Thomas. D. Your drinking doesnt sound that serious. 15. You are caring for a patient who has a long history of alcohol abuse. Recently, this patient went on a 5-day drinking binge, of which she has no memory. This is an example of: A. Selective memory. B. Wernickes syndrome. C. Blackout. D. Denial. 16. Nurses understand that in people who are addicted to alcohol, the person who is most responsible for the patients recovery is the: A. Psychiatrist. B. Nurse. C. AA sponsor. D. Patient. 17. You are socializing with a group of nurses who you work with on a routine basis. Terri is getting very loud and tells you that she usually has six or eight beers most evenings. She is defensive about your reaction to the amount of alcohol she consumes. She says, I have days where I cant remember what happened the night before, sure, but only once in a while. If you were Terris friend, the best action you could suggest to her would be: A. Maybe you should stop at four beers, Terri. B. I wont tell your husband what you told me. It sounds to me like your drinking is getting out of control. I cant continue to socialize with C. you if you continue to drink this much. D. Terri, I will call AA for you when you are ready. 18. Randi is a young model. She had been taking high doses of amphetamines to keep her weight down. She recently decided to cut back on the drugs and she is now experiencing amphetamine withdrawal. She presented to the clinic with which of the following sets of symptoms of amphetamine withdrawal? A. Chest pain, palpitations, and diaphoresis B. Depression, vivid dreams, and confusion C. Euphoria, hyperactivity, and hyperalertness D. Diaphoresis, clammy palms, and diarrhea 19. A young adult arrives in the after-hours clinic with dilated pupils, an elevated heart rate, extreme sensitivity to sounds around her, sense of being outside of his body, and fine tremors of the hands. The patient admits to recent use of an illegal street drug. As the nurse collecting this data, you suspect: A. LSD. B. Crack cocaine. C. Amphetamines. D. Downers. 20. Alcohol is a(n): A. Central nervous system (CNS) stimulant. B. CNS depressant. C. Antipsychotic. D. Antidepressant. 21. You are caring for a female patient who is a long-term alcoholic. She screams, Get the bugs off of my skin. I feel them all over my body! Get them off! She is experiencing what type of hallucination? A. Auditory B. Visual C. Taste D. Tactile 22. A 35-year-old female patient signs in as a voluntary commitment for treatment for drug abuse. She strongly maintains that he does not have a problem and states, Im only here because my boss threatened to fire me if I didnt come in. The best nursing response to this patient is: A. I wonder why your boss said that, if you dont have a problem. B. What happened that your boss threatened to fire you? C. Your boss sounds pretty harsh! D. Well, you are here on your own, so you can leave whenever you want to. 23. A patient has completed treatment for alcoholism. If treatment was successful, a nurse might expect which of the following outcome statements from this patient upon discharge? A. Now, if my family will just be good, I wont be back! B. I just know I can have an occasional drink and be fine. I know how to handle it now. C. I realize that Alcoholics Anonymous will always be a requirement. I am responsible for my own sobriety. D. I am so glad I found out what my problem is. I am cured now! 24. Which of the following is a club drug? A. Rohypnal B. Crack C. Placidyl D. Angel dust Completion Complete each statement. 25. You are completing the discharge plan for an alcoholic patient. Which support group should be included in the plan? Multiple Response Identify one or more choices that best complete the statement or answer the question. 26. You find out that your schizophrenic patient is also using opioids to counteract the frightening hallucinations. What term(s) is used to describe this? (Select all that apply) A. Dual diagnosis B. Bipolar schizophrenia C. Co-occurring disorder D. Opioid-related schizophrenia E. Schizophrenia: Addiction type 27. Which of the following are true about Alcoholics Anonymous? (Select all that apply) A. A physician referral is needed. B. Family and friends are encouraged to attend the regular meetings with the alcoholic. C. No last names are used at the meetings. D. The group for family members is Al-Anon. E. Offshoots such as Narcotics Anonymous follow a different philosophy. F. The individual is encouraged to admit he or she is powerless over alcohol. Answer Section MULTIPLE CHOICE 1. ANS: C These three symptoms are common with a stimulant such as cocaine. They could occur with the other substances but are not the hallmark symptoms. 2. ANS: A This medication is often initially used to treat alcohol withdrawal symptoms. Response B is a treatment for alcoholism not withdrawal. Response C is an antipsychotic and response D is used in opioid addiction. PTS: 1 3. ANS: B This response indicates the person is minimizing or not acknowledging that she has a problem. In the other responses the patient is acknowledging drinking but rationalizing the causes. 4. ANS: C Responses A, B, and D all reinforce use of ineffective defense mechanisms, including denial, rationalization, and minimization. Response C reflects understanding of the disease as a lifelong struggle. PTS: 1 5. ANS: B Blackouts are gaps in memory that are symptomatic of advanced alcoholism. 6. ANS: A Marijuana is a cannabinol. It is not a narcotic or an amphetamine. 7. ANS: C Disulfiram (Antabuse) is used to treat alcoholism by producing severe adverse effects in the presence of alcohol. Responses A and D are antipsychotics, and response B is an antianxiety medication used to treat alcohol withdrawal. 8. ANS: C This response supports a safe environment for this patient, the other patients, visitors, and staff. Ignoring him will escalate the situation. Because her judgment is impaired, trying to reason with her will be unsuccessful. 9. ANS: B This response shows that the wife is covering for her husband so he doesnt have to take responsibility for his actions. Response D is more denial. Responses A and C are indications of facing the reality of this situation. PTS: 1 10. ANS: B Supporting problem solving and helping her express herself is most important. Response A is advice giving, which is not appropriate. You dont have enough information to know if response C or D is correct. 11. ANS: B Withdrawal symptoms in a heavy drinker generally begin within 24 to 48 hours after the last drink, although they can occur as early as 8 hours after the last drink. 12. ANS: C This is the healthiest response that she is not accepting a victim role. In response A she continues to accept his bad behavior. In response B she is taking responsibility for his drinking, which is codependent. Response D views the meetings as escape rather than a place to work on issues. 13. ANS: A These are classic symptoms along with irritability, rhinorrhea, and chills. 14. ANS: A Response A is an open-ended statement to help her identify her feelings and encourage further exploration. Response B asks for him to understand why he feels this way and he may not know. Responses C and D reinforce denial. 15. ANS: C Blackouts are indications of advanced alcoholism when the person has no memory of recent events. 16. ANS: D Taking personal responsibility for the misuse of alcohol and the distress it has caused others is key to beginning recovery. It is the basis of Alcoholics Anonymous. Blaming others for ones problem may be an initial coping mechanism. PTS: 1 17. ANS: C Response C is reality based and avoids any enabling behavior that reinforce/support her drinking. The other responses are enabling. 18. ANS: B Withdrawal symptoms also include insomnia and lethargy. 19. ANS: A Hallucinogenics like LSD produce a sense of enhanced perception of the environment, which can contribute to a sense of depersonalization and panic. 20. ANS: B Though the initial reaction may be more like a stimulant, it is a CNS depressant. 21. ANS: D Tactile hallucinations are false perceptions of the sense of touch that often are described as something crawling on or under ones skin. Tactile hallucinations can occur as part of delirium tremens from long-term alcohol use and withdrawal. PTS: 1 22. ANS: B This response asks the patient to focus on reality rather than blaming others. This is the best response to challenge the probable denial. 23. ANS: C This response indicates the patient understands his or her personal responsibility to stay sober. The other responses show a lack of understanding that alcoholism is a lifelong disorder and that it is not caused by family problems. 24. ANS: A Rohypnal along with Ecstasy create disinhibition and amnesia. PTS: 1 COMPLETION 25. ANS: Alcoholics Anonymous Alcoholics Anonymous is the most accepted support group widely available throughout the world. PTS: 1 MULTIPLE RESPONSE 26. ANS: A, C Dual diagnosis, also known as co-occurring disorder, is very common in people with psychiatric disorders who self-medicate for uncomfortable symptoms. 27. ANS: C, D, F One of the strengths of this program is its anonymity, so last names are never used and meetings are closed. Admitting powerlessness over the substance abused is key to making progress. [Show More]

Last updated: 11 months ago

Preview 1 out of 284 pages

Reviews( 0 )

$26.00

Add to cart

Instant download

Can't find what you want? Try our AI powered Search

OR

GET ASSIGNMENT HELP
33
0

Document information


Connected school, study & course


About the document


Uploaded On

Aug 22, 2022

Number of pages

284

Written in

Seller


seller-icon
Nolan19

Member since 2 years

10 Documents Sold


Additional information

This document has been written for:

Uploaded

Aug 22, 2022

Downloads

 0

Views

 33

Document Keyword Tags

Recommended For You


$26.00
What is Browsegrades

In Browsegrades, a student can earn by offering help to other student. Students can help other students with materials by upploading their notes and earn money.

We are here to help

We're available through e-mail, Twitter, Facebook, and live chat.
 FAQ
 Questions? Leave a message!

Follow us on
 Twitter

Copyright © Browsegrades · High quality services·